You are on page 1of 62

The Institute of Chartered Accountants of Nepal

Suggested Answers of Advanced Accounting

CAP II Examination- December 2012

1. A, B and C carried on business in partnership, sharing profits and losses in the ratio of 1:2:3. They decided
to form a private limited company, AB (P) Ltd. and C is not interested to take over the shares in AB (P) Ltd. The
authorized share capital of the company is Rs. 1,200,000 divided into 12,000 ordinary shares of Rs. 100 each.
The company was incorporated and took over goodwill as valued and certain assets of the partnership firm on 31-03-
2012. The Balance Sheet of the partnership firm on that date was as follows:
______________________________________________________________
Liabilities Rs. Assets Rs.
Capital Accounts: PPE:
A 100,000 Machinery 120,000
B 200,000 Land 174,000
C 300,000 Motorcycles 30,000
Current Accounts: Furniture & fittings 11,000
A 39,420 Current Assets:
B 60,580 Stock 235,000
A‟s Loan A/c 28,000 Debtors 43,000
Add: Interest accrued 2,000 30,000 Cash in hand 87,000
Current Liability: C‟s overdrawn 100,000
Creditors 70,000
Total 800,000 Total 800,000
C, who retired was presented by the other partners (A and B) with one motorcycle valued in the books of the firm Rs.
9,000. The remaining motorcycles were sold in the open market for Rs. 13,000. C also received certain furniture for
which he was charged Rs. 2,000. The debtors which were all considered good, were taken over by C for Rs. 40,000.
A and B were charged in their profit sharing ratio for the book value of motorcycle presented by them to C.
It was agreed that C who is not willing to take the shares in AB (P) Ltd. was discharged first by providing necessary
cash. A and B should bring cash, if necessary.
AB (P) Ltd. took over the remaining furniture and fittings at a price of Rs. 13,000, the machinery for Rs. 125,000, the
stock at an agreed value of Rs. 200,000 and the land at its book value. The value of the goodwill of the partnership
firm was agreed at Rs. 88,000. The creditors of the firm were settled by the firm for Rs. 70,000. A‟s loan account
together with interest accrued was transferred to his capital account.
The purchase consideration was discharged by the company by the issue of equal number of fully paid up equity
shares at par to A and B.
Prepare realization account, capital account of the partners and cash account. Also draw the balance sheet of AB (P)
Ltd. 20
Answer:
Dr. Realization Account Cr.
Particulars (Rs.) Particulars (Rs.)-
_____________________________________________________________________________
To Machinery 1,20,000 By Creditors 70,000
To Land 1,74,000 By AB (P) Ltd.-Purchasse
Consideration 6,00,000
(Refer Working Note)
To Motorcycles 30,000 By A‟s Capital A/c 3,000
To Furniture & fittings 11,000 By B‟s Capital A/c 6,000
To Stock 2 ,35,000 By C‟s Capital A/c 42,000
(2000 + 40,000)
To Debtors 43,000 By Cash (Sale of Motorcycle) 13,000
To Cash (payment of creditors) 70,000
To Profit transferred to:
A‟s Capital A/c 8,500
B‟s Capital A/c 17,000
C‟s Capital A/c 25,500

Total 7,34,000 Total 7,34,000

Dr. Partners‟ Capital Accounts Cr.


_____________________________________________________________________________________
A B C A B C
Suggested Answers – Advanced Accounting
CAP II Examination – December 2012
To Current A/c - - 100,000 By Balance b/d 100,000 200,000
3,00,000
To Realisation A/c 3,000 6,000 42,000 By Current A/c 39,420 60,580 -
(Assets taken over)

To Equity Shares in
AB (P) Ltd. 300,000 300,000 - By A‟s Loan 30,000 - -
To Cash - - 183.500 By Realisation A/c 8,500 17,000 25,500
(Realization Profit)
By Cash A/c 125,080 28,420 -

Total 303,000 306,000 3,25,500 Total 303,000 306,000 325,500

Dr. Cash Account Cr.


Particulars (Rs.) Particulars (Rs.)-
_________________________________________________________________________________
To Balance b/d 87,000 By Realisation A/c 70,000
To Realisation A/c 13,000 By C‟s Capital A/c 183,500
To A‟s Capital A/c 125,080
To B‟s Capital A/c 28,420
Total 253,500 Total 2,53,500

Balance Sheet of AB (P) Ltd.


_____________________________________________________________________________
Liabilities Rs. Assets Rs.
Authorized Share Capital PPE:
12,000 Equity Shares of Rs. 100
each 1,200,000 Goodwill 88,000
Issued, Subscribed & Paid up: Land 174,000
6,000 Equity Shares of Rs. 100 Machinery 125,000
each fully paid up (shares were Furniture &Fixture 13,000
issued for consideration otherwise Current Assets:
than for cash) Stock 200,000
600,000
Total 600,000 Total 600,000

Working Note:

Calculation of Purchase Consideration

Assets taken over by AB (P) Ltd. (Rs.)


Machinery 125,000
Furniture & fittings 13,000
Land 174,000
Stock 200,000
Goodwill 88,000
Purchase Consideration 600,000

Purchase consideration is discharged by the issue of equal number of equity shares of Rs. 100 each (3,000 shares) at
par to A & B.

2.
a) ABC Fertilizers presents the following balance sheets as on 31-03-2069 and 32-03-2068. You are
required to prepare cash flow statement: 10

(Rs in '000)
Balance Sheet 31-03-2069 32-03-2068
Equity Share Capital 8,500 7,000
General Reserve 3,800 4,000
Profit & Loss Account - 250
Share Premium Account 1,500 750
Share Holders Fund 13,800 12,000

The Institute of Chartered Accountants of Nepal


2 of 62
Suggested Answers – Advanced Accounting
CAP II Examination – December 2012
Secured Loans 4,800 5,000
Unsecured Loans 5,350 4,000
Loan Funds 10,150 9,000
Total Sources 23,950 21,000
PPE
Gross Block 22,400 21,000
Less: Accumulated Depreciation 3,450 3,200
Net Block 18,950 17,800
Capital Work In Progress 1,860 -
Investments 1,650 2,320
Current Asset, Loans & Advances
Inventories 2,510 2,600
Debtors 1,090 1,200
Cash & Bank Balance 120 280
Loans 1,700 200
Advance Tax - 500
(A) 5,420 4,780
Less Creditors 1,050 1,200
Outstanding Expense 30 -
Tax Provision - 500
Proposed Dividend 3,400 2,800
(B) 4,480 4,500
Net Current Asset 940 280
Miscellaneous Expenditure 550 600
Total Applications 23,950 21,000

Other Information:
i) PPE costing Rs. 400,000 (accumulated depreciation Rs. 300,000) were sold for Rs. 150,000.
ii) Actual tax liability for fiscal year 2067-068 was Rs. 500,000.
iii) Loan represents loan given to other companies.
iv) Interest on loan funds for fiscal year 2068-069 was at Rs. 1,421,000 and interest & dividend income were
Rs. 402,000.
v) Investments costing Rs. 2,000,000 were sold for 2,500,000.

b) Prischal sells goods on hire-purchase at cost plus 25%, prepare hire purchase trading account from
the information given below: 10
Particulars Rs.
Stock with Customers on Hire -Purchase (Opening) 162,000
Stock in hand at shop (Opening) 324,000
Instalments Overdue (Opening) 150,000
Purchases during the year 1,080,000
Goods Repossessed (Instalments not due Rs. 36,000) 9,000
Stock at shop excluding repossessed goods (Closing) 360,000
Cash received during the year 1,035,000
Instalments Overdue (Closing) 162,000
The Vendor Spent Rs. 2,000 on goods repossessed and then sold it for Rs. 15,000.

Answer:
a)
CASH FLOW STATEMENT
(Rs in ‘000)
Cash Flow from Operation Activities Rs.
Change in General Reserve -200
Change in Profit & Loss Account -250
Proposed Dividend 3400
Profit Before Tax 2950
Add: Depreciation 550
Add: Miscellaneous Exp 50
Less: Profit on Sale of PPE -50
Less: Profit on Sale of Investments -500 50
Fund Flow from Operations 3000
The Institute of Chartered Accountants of Nepal
3 of 62
Suggested Answers – Advanced Accounting
CAP II Examination – December 2012
Add: Interest Paid 1421
Less: Interest & Dividend Received -402
Add/(Less): Changes in Working Capital
Inventories 90
Debtors 110
Creditors -150
Outstanding Expense 30 80
Cash Flow from Operation Activities (Before Tax) 4099
Less: Advance Tax for 2068-2069 0
Cash Flow from Operation Activities (After Tax) A 4099
Cash Flow from Financing Activities
Issue of Shares 2250
Repayment of Secured Loans -200
Raising of Unsecured Loans 1350
Interest Payment -1421
Dividend Payment -2800
(B) -821
Cash Flow from Investment Activities
Purchase of PPE -1800
Sale of PPE 150
Capital WIP -1860
PPE(NET) -3510
Purchase of Investments -1330
Sale of Investments 2500
Investments (Net) 1170
Loans -1500
Interest & Dividend Income 402
C -3438
Increase/ Decrease in Cash & Bank Balance (A+B+C) -160
Add: Opening Cash & Bank Balance 280
Closing Cash & Bank Balance 120

b. Hire Purchase Trading Account

Particulars Rs. Particulars Rs.

To Opening Balance By Cash Received (Instalments) 1,035,000


Hire Purchase Debtors 150,000 By Stock Reserve (Opening ) W.N. 2 32,400
Hire Purchase Stock (Instalments 162,000 By Goods sold on Hire Purchase (Loading)
Overdue) (W.N.1) 261,000
By Cash Received (On sale of Re-possessed
goods) 15,000
To Goods sold on hire Purchase
(W.N.1) 1,305,000 By Closing Balance
Hire Purchase Stock (Inst. Overdue) 384,000
To Cash (Expense) 2,000 (W.N.4)

To Stock reserve (W.N.5) 76,800 Hire Purchase Debtors 162,000


To Profit & Loss Account 193,600
1,889,400 1,889,400

Working Notes
1.
Memorandum Stock at Shop Account
Particulars Rs Particulars Rs
To Balance B/d 324,000 By Goods Sold on Hire 1,044,000
Purchase Account

To Purchases(at Cost) 1,080,000 By Balance C/d 360,000

1,404,000 1,404,000

The Institute of Chartered Accountants of Nepal


4 of 62
Suggested Answers – Advanced Accounting
CAP II Examination – December 2012

Goods Sold on Hire Purchase Account at Invoice Price (1044000 x 125%) 1,305,000
Loading Rs 1305000-1044000 261,000

2.
Opening Reserve Account
162,000x25/125=32,400

3.
Hire Purchase Debtor Account
Particulars Rs Particulars Rs
To Balance B/d 150,000 By Cash Received 1,035,000
To Goods Sold on Hire Purchase By Hire Purchase Stock Account (Bal
1,305,000 .Fig) 258,000

By Balance c/d 162,000


1,455,000 1,455,000

4.
Hire Purchase Stock Account
Particulars Rs Particulars Rs
To Balance B/d 162,000 By Goods Repossessed 36,000
To Hire Purchase Debtors 258,000 By Balance C/d (Bal Fig) 384,000

420,000 420,000

5.
Closing Stock Reserve Account
384,000x25/125=76,800

3.
a) Gajani Limited operates a number of retail outlets to which goods are invoiced at wholesale price which is
cost plus 25%. These outlets sell the goods at the retail price which is wholesale price plus 20%.

Following is the information regarding one of the outlets for the year ended 21-3-2012:

Particulars Rs.
Stock at the outlet 1-4-2011 30,000
Goods invoiced to the outlet during the year 324,000
Gross profit made by the outlet 60,000
Goods lost by fire ?
Expenses of the outlet for the year 20,000
Stock at the outlet 31-3-2012 36,000

You are required to prepare the following accounts in the books of Gajani Limited for the year ended 31-3-2012: 10

i) Outlet Stock Account


ii) Outlet Profit & Loss Account
iii) Stock Reserve Account

b)
i) Mr. Gee started work on 1st January 2011 on a contract for Rs. 500,000. On 31st March 2011 when he
prepared his final accounts, the following information relating to the contract was extracted from his books of
account:
Rs.
Materials issued from stores and sent to site 160,000

The Institute of Chartered Accountants of Nepal


5 of 62
Suggested Answers – Advanced Accounting
CAP II Examination – December 2012
Wages Paid 101,200
Wages outstanding on 31st March 2011 37,520
New machine purchased and sent to site on 1st January 2011 148,000
Direct charges paid 7,500
Direct charges outstanding on 31st March 2011 600
Establishment charges apportioned to contract 6,400

On 31st March 2011 materials lying unused at site were valued at Rs. 21,620. Machines were depreciated @ 20% per
annum. Value of work certified by 31st March 2011 was Rs. 350,000 while the cost of work done but not yet certified
as on that date was Rs. 18,000. On the basis of architect‟s certificate Mr. Gee had received a total sum of Rs. 280,000
from the contractee till 31st March 2011.
You are requested to prepare contract account in Mr. Gee‟s ledger with appropriate working notes. 7

ii) X Ltd. takes over Y Ltd. on Shrawan 01, 2069 and discharges consideration for the business as
follows:
a. Issued 42,000 fully paid equity shares of Rs. 10 each at par to the equity share holders of Y Ltd.
b. Issued fully paid up 15% preference shares of Rs. 100 each to discharge the preference shareholders
(Rs. 170,000) of Y Ltd. at a premium of 10%.
c. It is agreed that the debentures of Y Ltd. (Rs. 50,000) will be converted into equal number and
amount of 13% debentures of X Ltd.
Calculate the purchase consideration of X Ltd. 3

Answer:
a)
Dr. Outlet Stock Account Cr.
Particulars (Rs.) Particulars (Rs.)-
_________________________________________________________________________________
To Balance b/d 30,000 By Sales (Working Note 1) 360,000
To Goods sent to outlet 324,000 By Goods Lost by fire 18,000
To Gross Profit c/d 60,000 By Balance c/d 36,000
Total 414,000 Total 414,000

Dr. Outlet Profit & Loss Account Cr.


Particulars (Rs.) Particulars (Rs.)-
_________________________________________________________________________________
To Expenses 20,000 By Gross Profit b/d 60,000
To Goods Lost by fire 18,000
(Working Note 2)
To Profit Transferred 22,000
Total 60,000 Total 60,000

Dr. Stock Reserve Account Cr.


Particulars (Rs.) Particulars (Rs.)-
_________________________________________________________________________________
To Profit & Loss A/c – Transfer 6,000 By Balance b/d 6,000
To Balance c/d 7,200 By Head Office- Profit & Loss A/c
(Working Note 3)
(Stock Reserve Required) 7,200
Total 13,200 Total 13,200

Working Notes:

(1) Wholesale Price 100+25 = 125


Retail Price 125+20% = 150
Gross Profit at the outlet
Wholesale Price – Retail Price (150-125) = 25
Retail Sales Value = 60,000 x 150/25 = 3,60,000

(2) Goods lost by fire


= Opening Stock + Goods Sent + Gross Profit – Sales – Closing Stock
The Institute of Chartered Accountants of Nepal
6 of 62
Suggested Answers – Advanced Accounting
CAP II Examination – December 2012
= 30,000 + 3,24,000 + 60,000 – 3,60,000 – 36,000
= Rs. 18,000

(3) Stock Reserve


On Opening Stock = 30,000 x 25/125 = Rs. 6,000

On Closing Stock = 36,000 x 25/125 = Rs. 7,200

b) i)
In Gee’s Ledger
Contract Account
Particulars Rs. Particulars Rs.
To Materials sent to site 160,000 By Unused materials at site as on 31st
To Wages paid 101,200 March 2011 21,620
Add: Outstanding wages 37,520 138,720 By Machines (written down value as on
To Machines (Cost) 148,000 31st March 2011) 140,600
To Direct Charges 7,500 By Work in progress:
Add: Outstanding DC 600 8,100 Certified 350,000
To Establishment Charges 6,400 Uncertified 18,000 368,000
To Profit & Loss A/c 36,800
To Balance c/d 32,200
530,220 530,220
To Unused Materials at site 21,620
To Machines 140,600
To Work in progress:
Certified 350,000
Uncertified 18,000
368,000
Less: Balance b/d 32,200 335,800

Working Notes:
i) Calculation of written down value of machines on 31 st March 2011:
Cost of Machine on 1st January 2011 148,000
Less: Depreciation on NPR 148,000 @20% p.a. for 3 months
= NPR 148,000 x 20/100 x 3/12 7,400
140,600
ii) Calculation of amount to be transferred from Contract Account to Profit & Loss Account:
Total surplus in Contract Account as on 31st March 2011
= NPR 21,620 + NPR 140,600 + NPR 368,000 – NPR 160,000 – NPR 138,720 – NPR 148,000 – NPR 8,100 – NPR
6,400 = NPR 69,000.

Profit to be credited to Profit & Loss Account


= Notional Profit 

= NPR 69,000 x 2/3 x 280,000/350,000 = NPR 36,800

b) ii)
Particulars Rs.
Equity Shares(42000*10) 420,000
Preference Share Capital 170,000
Add: Premium On redemption 17,000 187,000
Purchase Consideration 607,000

Note: There is no relevance of (c)

4.
a) A company issues 1000, 14% of Rs. 1,000 each at a premium of 20%. 60 % of the issue was underwritten
by Mr. Akash Chettri & Co. for a commission @ 1.50% of the issue price of debentures underwritten.
The Institute of Chartered Accountants of Nepal
7 of 62
Suggested Answers – Advanced Accounting
CAP II Examination – December 2012
Applications were received for 800 debentures which were accepted and payments were received in full. Give the
journal entries. 5

b) Following are the extracts from the record of Alpha Bank Ltd. in respect of the year ending Ashadh 31,
2069:

On Shrawan 01, 2068; bills for collection were Rs. 400,000. During 2068-069 bill received for collection amounted
to Rs. 6,050,000, bills collected were Rs. 4,500,000 and bills dishonored and returned were Rs. 300,000.
Prepare bills for collection (Assets) account and bills for collection (Liability) account. 5
c) GP Ltd. received a grant of Rs. 40 lakhs from the Government for the purpose of special machinery during
fiscal year 2064-065. The cost of machinery was Rs. 400 lakhs and had a useful life of 10 years. During fiscal
year 2068-069, the grant has become refundable due to non-fulfillment of certain conditions attached to it. Assuming
the entire grant was deducted from the cost of machinery in the year of acquisition; state with reason, the accounting
treatment to be followed in the fiscal year 2068-069. 5

Answer:
a)
The Journal Entries are as follows.

S.N. Particulars Debit Credit


1 Bank Account 960,000
To 14% Debentures 800,000
To Premium on Issue of Debentures 160,000
Allotment of 800 debentures @Rs 1200 as per Directors Resolution of ……………
2 Akash Chettri & Co 144,000
To 14% Debentures 120,000
To Premium on Issue of Debentures 24,000

Allotment of 120 debentures to the Underwriters in pursuance of their agreement dated………..


3 Underwriting Commission Account 10,800
To Akash Chettri & Co 10,800
Commission @1.5% on 600 Debentures at the issue price of Rs 1200
4 Bank 133,200
To Akash Chettri & Co 133,200
Amount Received form the underwriter from settlement

Note: Since Akash Chettri & Co underwrite 60% of the issue , the company must itself be treated as an
underwriter of the remaining 40% . In the absence of any information, the applications for 800 debentures must be
deemed to have marked 60% in favour of underwriters i.e. 480 debentures. The underwriters are, therefore, liable
to take up 120 debentures (600-480).

b)
Bills for Collection (Assets) A/c
Date Particulars Rs. Date Particulars Rs.
1/4/2068 To balance b/d 400,000 2068/069 By Bills for collection
2068/069 To Bills for collection (Liabilities) A/c 4,500,000
( Liabilities) A/c 6,050,000 By Bills for collection
(Liabilities) A/c 300,000
31/3/2069 By Balance c/d 1,650,000
6,450,000 6,450,000

Bills for Collection (Liabilities) A/c


Date Particulars Rs. Date Particulars Rs.
2068/069 To Bills for collection 1/4/2068 By Balance b/d 400,000

The Institute of Chartered Accountants of Nepal


8 of 62
Suggested Answers – Advanced Accounting
CAP II Examination – December 2012
( Assets) A/c 4,500,000 2068/069 By Bills for collection
To Bills for collection (Assets ) A/c 6,050,000
( Assets) A/c 300,000
31/3/2069 To Balance c/d 1,650,000
6,450,000 6,450,000

c) As per para 32 of NAS 10 Accounting for Government Grants, in case of refund of government grant related
to a specific fixed asset shall be recorded by increasing the carrying amount of the asset. The cumulative additional
depreciation that would have been recognized to date as an expense in the absence of the grant shall be recognized
immediately as an expense.

The computation of depreciation and expense can be given as:


Cost of Machinery Rs. 400 lacs
Less: Grant Received Rs. 40 lacs
Cost of Machinery Rs. 360 lacs
Useful life of Machinery 10 years
Depreciation per year as per straight line method (Assuming residual value to be zero Rs. 360 lacs/10 = Rs. 36 lacs
Total depreciation for 4 years (2064/65 to 2067/68) = 36×4 =Rs. 144 lacs

Cost of machinery without government grants Rs. 400 lacs


Useful life of Machinery 10 years
Depreciation that would have been recognized in absence of grant Rs. 400/10 = 40 Lacs
Cumulative total depreciation that would have been recognized in absence of grant=40×4 =160 lacs
Expenses to be recognized in FY 2068/069 = 160 lacs – 144 lacs=16 lacs

Depreciation to be charge for another 6 years = (400 – 160)/6


Amount of depreciation as an expense for 2068/069 = 40 + 16 = 56 lacs
Depreciation for remaining 5 years = 40 lacs per year

5.
a) While preparing its final accounts for the year ended 31 st March, 2012 Sky Limited created a provision for
bad and doubtful debts are 2% on trade debtors. A few weeks later the company found that payments from some of
the major debtors were not forthcoming. Consequently the company decided to increase the provision by 10% on the
debtors as on 31st March, 2012 as the accounts were still open awaiting approval of the Board of Directors. Is this to
be considered as an extraordinary item or prior period item? The company wants to treat the expenditure as deferred
revenue expenditure. Give your comments for the financial year ending on 31-03-2012 in the context of relevant
NAS. 5

b) A company obtained term loan during the year ended 31 st March, 2012 to an extent of Rs. 650 lakhs for
modernization and development of its factory. Building worth Rs. 120 lakhs were completed and plant and machinery
worth Rs. 350 lakhs were installed by 31st March, 2012. A sum of Rs. 70 lakhs has been advanced for assets the
installation of which is expected in the following year. Rs. 110 lakhs has been utilized for working capital
requirements. Interest paid on the loan of Rs. 650 lakhs during the fiscal year 2011-012 amounted to Rs. 58.50 lakhs.
How should the interest amount be treated in the account of the company? Give your comments for the financial year
ending on 31-03-2012 in the context of relevant NAS. 5
c) A pharma company spent Rs. 135 lakhs during the accounting year ended 31 st March, 2012 on a research
project to develop a drug to treat “AIDS”. Experts are of the view that it may take four years to establish whether the
drug will be effective or not and even if found effective it may take two to three more years to produce the medicine,
which can be marketed. The company wants to treat the expenditure as deferred revenue expenditure. Give your
comments for the fiscal year ending on 31-03-2012 in the context of relevant NAS. 5

Answer:
a)
As per NAS 2, revision of an estimate does not bring the resulting amount within the definition either of prior item or
of an extraordinary item.

In the given case, Sky Limited created a provision for bad and doubtful debts at 2 % on trade debtors while preparing
its final accounts for the year ended 31st March, 2012. Subsequently, the company decided to increase the provision
by 10%. As per NAS 2, this change in estimate is neither a prior period item nor an extraordinary item.

The Institute of Chartered Accountants of Nepal


9 of 62
Suggested Answers – Advanced Accounting
CAP II Examination – December 2012
However, as per NAS 2, a change in accounting estimate, which has a material effect in the current period, should be
disclosed and quantified. Any change in an accounting estimate, which is expected to have a material effect in later
periods should also be disclosed.

The expenditure of revenue nature may be treated as deferred revenue expenditure if its benefit is likely to be derived
over a number of years and alternatively, if there is a huge loss due to unforeseen circumstances. However, increase
in provision after the balance sheet date in the given case being an accounting estimate cannot be treated as deferred
revenue expenditure.

b)

According to NAS 8 (“Borrowing Cost”), interest on borrowed funds, which is directly related to the acquisition,
construction or production of qualifying asset should be capitalized. As factory building, Plant and Machinery are
qualifying asses as per NAS 8 interest paid on the loan being borrowing cost should be capitalized and included in the
gross book value of these assets. The interest pertaining to the money spent on the working capital should be charged
off to the Profit and Loss Account. In the given case, the interest amount of Rs. 58.50 lakhs shall be treated as
follows:

Proportion Rs. In lakhs


To be added to the cost of building 120  58.50 10.80
650
To be added to the cost of Plant and Machinery 350  58.50 31.5
650
To be added to the cost of work in progress 70  58.50 6.3
650

To be added to the cost of Profit and Loss A/c 110  58.50 9.9
650

Total 58.50

c)
As per NAS 27 on intangible assets, the research cost be expenses as and when incurred, in other words the cost of
research cannot be capitalized. The intangible asset arising from research should not be recorded as an asset and
therefore the research cost of internal project shall be treated as an expense in financial statement.

The development expenses, cost of internal project also to be expenses as incurred unless they meet asset recognition
criteria, before recognizing these costs as assets the following points should be demonstrated:

 Technical feasibility of the product


 Availability of product for use or sale
 Identification of cost incurred
 Probability of external market or
 The realistic expectation that there will be sufficient revenues to cover cost.

In the given case, the above conditions not having been fulfilled (Nothing is stated about future revenue or benefits),
the sum of Rs. 135 lakhs should be charged to the profit and loss account as an expense in the accounting year ended
31st March, 2012.

6. Write short notes on any four of the following: (4×2.5=10)


a) Mention any four areas in which different accounting policies may be adopted by different enterprises.
b) Contingency reserve
c) Non-performing assets
d) Conditions to be satisfied to capitalize the borrowing costs
e) Materiality and prudence

Answer:
a)
Major areas in which different accounting policies may be adopted by different enterprise includes:

 Methods of depreciation, depletion and amortization, e.g., WDV method, SLM method
The Institute of Chartered Accountants of Nepal
10 of 62
Suggested Answers – Advanced Accounting
CAP II Examination – December 2012
 Treatment of expenditure during construction, e.g. capitalization, written off, deferment
 Conversion or translation of foreign currency items, e.g. average rate , TT buying rate
 Valuation of inventories, e.g., FIFO, Weighted average method
 Treatment of goodwill, e.g., capitalization method, super profit method
 Valuation of Investments, e.g. lower of cost and fair value

b)
The Contingency reserves are sum set aside to cover anticipated future liabilities or reduction in assets value. This
reserve is required when the company believes the value of its assets likely decrease or it has incurred liabilities and it
is able to reasonable estimate the amounts loss. Contingency reserves are net up by deducting the appropriate sum
from income. Contingencies include:
 Potentially uncollectable money owed to company.
 Potential obligation under product warranties or related to products defects judgment for pending threaten
litigation.
 Likely loss due to fire and other hazards.
The Contingency reserve must be disclosed in financial statement when required and may be utilized for the
following purposes:
a. Expenses or loss of profits arising out of accidents, strikes or circumstances which the management could
not have prevented.
b. Expenses on replacement or removal of plant or works (other than normal maintenance or renewals).
c. Statutory obligation for payment of any compensation, if there is no special provision for such
compensation.

c)
While preparing financial statements of a bank, it is necessary to identify non-performing assets mostly based on
statutory/regulatory norms. An asset becomes non-performing when installment of matured principal and or income
from it is not received by the bank for a certain period. Income from non-performing assets can only be accounted for
as and when it is actually received. Nepal Rastra Bank has issued directives for the classification of loans and
advances. Necessary provision should be made for non-performing assets classifying as sub-standard, doubtful or loss
assets as the case may be as per the rate prescribed by Nepal Rastra Bank.

d)
The following conditions should be satisfied for capitalization of borrowing costs:
a) Those borrowing costs, which are directly attributable to the acquisition, construction or production of
qualifying asset, are eligible for capitalization. Directly attributable costs are those costs that would have been
avoided if the expenditure on the qualifying asset had not been made.
b) Qualifying assets will give future economic benefit to the enterprise and the cost can be measured reliably.

e)
Materiality:
Information is material if its omission or misstatement could influence the economic decisions of users taken on the
basis of the financial statements. It depends on the size of the item or error judged in the particular circumstances of
its omission or misstatement. Often separate line item or sub-item is decided bases on materiality. National level law
may specify materiality limit for separate disclosure of an item.

Prudence:
Prudence is the inclusion of a degree of caution in the exercise of the judgments needed in the making the estimates
required under conditions of uncertainty, such that assets or income are not overstated and liabilities or expenses are
not understated. The exercise of prudence does not allow, for example, the creation of hidden reserves or excessive
provisions.

The Institute of Chartered Accountants of Nepal


11 of 62
The Institute of Chartered Accountants of Nepal
Suggested Answers of Audit and Assurance

CAP II Examination- December 2012

1. As an auditor, give your opinions with reasons on the following cases: (45=20)

a) Your client has computed the deferred tax assets or liabilities on: i. Fixed assets as per financial base Rs.
500,000, as per tax base Rs. 600,000; ii. Other assets as per financial base 300,000 as per tax base Rs. 200,000 and other
liabilities as per financial base Rs. 100,000 as per tax base Rs. 50,000. Consider applicable tax rate 5% and opening
balance of deferred tax asset is Rs.5,000As a Auditor how would be check the accounting treatment made on deferred
tax by your client (show your computation as well)?

b) Puran (A practicing Chartered Accountant) has been appointed as statutory auditor of ABC Development Bank
Limited for the fiscal year 2068/69. Puran has been holding certain paid up share capital of the bank since the year 2060.
Is the appointment of Puran is valid?

c) New auditor of XYZ Limited signed the financial statements for the year 2068/69 without considering the
previous year auditor's report and financial figures.

d) The Balance sheet of ABC Ltd. includes inventory amounting to Rs. 3 crores out of total assets of 20 crores. The
inventories were valued at cost. The market price of the inventories was Rs. 2.5 crores. The company has disclosed this
fact in the notes to accounts.

Answer:
a) Nepal Accounting Standard 9 (NAS) defined on deferred tax assets and liabilities. The NAS suggested for
computing deferred tax assets or liabilities on temporary differences on tax base and financial base and booking
corresponding income or expenses. The computation of deferred tax has been as follows:

Particulars Financial Tax Base (B) Deferred (Assets) Applicable Deferred Tax (Assets)
Base (A) /Liabilities C= (A-B) /Liabilities @ 5% D= C*5%
1.Fixed Assets 500,000 600,000 (100,000) Yes (5,000)
2.Other Assets 300,000 200,000 100,000 Yes 5,000
3. Liabilities 100,000 50,000 (50,000) Yes (2,500)
Net Asset (2,500)
Previous year Asset Balance (5,000)
Decrease in Asset (Charged to 900,000 850,000 (50,000) (2,500)
PL)

Accordingly closing balance of deferred tax should be Rs. 2,500 and deferred tax asset should be credited by Rs. 2,500
by charging corresponding amount to profit & loss account as deferred tax expenses.

b) Section 112 of the Companies Act, 2063 stated the disqualifications of auditor: As per Act None of the certain persons
or the firms or companies in which such persons are partners shall be qualified for appointment as auditor and shall,
despite appointment as auditor, continue to hold office.

Provision regarding appointing shareholder as auditor as per Companies Act stated that: persons or the firms or
companies who is substantial shareholder of the company or a shareholder holding one percent or more of the paid up
capital of the company or his close relative cannot be appointed as auditor.

In the instance case the % of holding paid up capital is not mentioned, if the Puran himself or firm or company he is
partner or shareholder or his close relative do not hold one percent or more than one percent of the paid up capital of
ABC Development Bank Limited only than Puran is qualified to be appointed as auditor.

c) NSA 24 Para 9 has defined the treatment of the previous figures and auditors' report. As per the Para 9, the auditor
should obtain sufficient appropriate audit evidence that the corresponding figures meet the requirement of the relevant
financial reporting framework. The extent of audit procedures performed on the corresponding figures is significantly
less than for the audit of the current period figures and is ordinarily limited to ensuring that the corresponding figures
have been correctly reported and are appropriately classified. This involves the auditor assessing whether:
i) accounting policies used for the corresponding figures are consistent with those of the current period or whether
appropriate adjustments and/or disclosures have been made; and
ii) corresponding figures agree with the amounts and other disclosures presented in the prior period or whether
appropriate adjustments and/or disclosures have been made.
Suggested Answers – Audit and Assurance
CAP II Examination – December 2012
In Para 10 when the financial statements of the prior period have been audited by another auditor, the incoming auditor
assesses whether the corresponding figures meet the conditions specified in paragraph 9 above and also follows the
guidance in NSA 21, “Initial Engagements-Opening Balances.”
In Para 11 when the financial statements of the prior period were not audited, the incoming auditor nonetheless assesses
whether the corresponding figures meet the conditions specified in paragraph 9 above and also follows the guidance in
NSA 21.
In Para 12 if the auditor becomes aware of a possible material misstatement in the corresponding figures when
performing the current period audit, the auditor performs such additional procedures as are appropriate in the
circumstances.
When the comparatives are presented as corresponding figures, the auditor should issue an auditor‟s report in which the
comparatives are not specifically identified because the audit opinion is on the current period financial statements as a
whole, including the corresponding figures.
When the auditor‟s report on the prior period, as previously issued, included a qualified opinion, disclaimer of opinion,
or adverse opinion and the matter which gave rise to the modification is: (a) unresolved, and results in a modification of
the auditor‟s report regarding the current period figures, the auditor‟s report should also be modified regarding the
corresponding figures; or (b) unresolved, but does not result in a modification of the auditor‟s report regarding the
current period figures, the auditor‟s report should be modified regarding the corresponding figures.

d) As per Nepal Accounting Standard 4, Inventories should be measured at lower of cost or net realizable value. In the
present case the cost price of the inventories is 3 crores and net realizable value is 2.5 crores and hence the inventories
should be presented at 2.5 crores in the balance sheet. However, the company has presented the inventories at 3 crores
and disclosed in the notes to accounts that the inventories have been presented at cost although its net realizable value is
lower than the cost. Mere disclosure of this fact in the notes however does not result into compliance with the accounting
standard. Hence as an auditor I will qualify my audit report because inventory in the present case represents material
item of the assets of the company and it has been materially misstated in the balance sheet.

2. Answer the following:


a) You have taken up the assignment for Due Diligence Review of the financial statement as of 31 Asadh 2069 of Das
Dhunga Bank Ltd for facilitating the merger process with Lucky Finance Co. Ltd. Write down any seven points you have
to pay due consideration on carrying out the review on assets side (covering each asset category) of the financial
statement. 7

b) As a quality control manager of the audit engagement team of D Limited, how can you ensure that your team has
performed the audit works as per the standard maintained by your audit firm along with the other mandatory compliance
of various national and international standards in auditing and accounting? Points out the review procedures to ensure the
quality of the audit work? 8

Answer:
a) Upon taking up the assignment for due diligence review of the Das Dhunga Bank Ltd; i have to pay due
following due consideration on review assets side of the financial statement:
i. Loan & Advances: The due consideration to be paid on loan and advances is whether the adequate loan loss
provision has been made in line with Nepal Rastra Banks Directive applicable to the bank.
ii. Investment: The due consideration to be paid on investment is that whether the investment has been made in
line with Nepal Rastra Banks Directive in eligible portfolio. If investment is doubtful of recovery to see whether
necessary provision has been set aside or otherwise. Further in applicable cases have to see whether required
reserves/funds against the investment been set aside.
iii. Fixed Assets: The due consideration to be paid on fixed assets is that whether all the fixed assets appearing in
financial statement legally owned by the entity, physically available and are in working condition. The major fixed assets
should be revalued with the help of expert for the purposes of DDA.
iv. Advance Tax: The due consideration to be paid on Advance Tax is to see whether the amount shown as
advance tax are really an existed asset representing the cash deposits made and advance tax made by parties and are not
time barred for claiming for assessment of tax purposes.
v. Non-Banking Assets (NBA): To see whether the NBA are on the possession and ownership of the bank. See
whether disposal (sales) process of NBA has not been suffered any hindrances like NBA is insufficient for covering
dues recoverable, NBA is subject to lien of others , NBA is under legal cases. Further should be ensured that NBA is
adequately provisioned as per NRB Directives.
vi. Advances: To see the recoverability of advances. If advances are doubtful of recoverability, ensure that
whether adequate provision against possible loss has been made.
vii. Deposits : To see the existence of various deposits . If deposits are doubtful of refund and are not backed
up with sufficient documents, ensure that whether adequate provision against possible loss has been made.

b) The quality control manager inform the responsibilities of the engagement team and also give some of the things to
be ensured before starting the works as follows;
 understanding the nature of the entity's business
 Possible risk related issues
 Problems that may arise; and
The Institute of Chartered Accountants of Nepal
2 of 62
Suggested Answers – Audit and Assurance
CAP II Examination – December 2012

 The detailed approach to the performance of the engagement.


The engagement team's responsibilities includes maintaining an objective state of mind and an appropriate level of
professional skepticism, and performing the work delegated to them in accordance with the ethical principle of due care.

The quality control manager before review the files, will supervise the engagement team and do the following works:
 Tracking the progress of the audit engagement;
 Considering the capabilities and competence of individual members of the engagement team, whether they have
sufficient time to carry out their work, whether they understand their instructions, and whether the work is being carried
out in accordance with the planned approach to the audit engagement.
 Addressing significant issues arising during the audit engagement, considering their significance and modifying the
planned approach appropriately.
 Identifying matters for consultation or consideration by more experienced engagement team member during the
audit engagement.

The quality control manager will review the work performed by their team members and consider the following while
reviewing the works and working papers:
 The work has been performed in accordance with professional standards and regulatory and legal requirements;
 Significant matters have been raised for further consideration;
 Appropriate consultations have taken place and the resulting conclusions have been documented and implemented.
 There is a need to revise the nature, timing and extent of work performed;
 The work performed supports the conclusions reached and is appropriately documents;
 The evidence obtained is sufficient and appropriate to support the auditor's report;
 The objectives of the engagement procedures have been achieved.

The engagement partner conducts timely reviews at appropriate stages during the engagement. This allows significant
matters to be resolved on a timely basis to the engagement partner‟s satisfaction before the auditor‟s report is issued. The
reviews cover critical areas of judgment, especially those relating to difficult or contentious matters identified during the
course of the engagement, significant risks, and other areas the engagement partner considers important. The engagement
partner need not review all audit documentation. However, the partner documents the extent and timing of the reviews.
Issues arising from the reviews are resolved to the satisfaction of the engagement partner.

3. Giv
e your comments on the following: (35=15)

a) Sahara Garment Private Limited is one of the leading garments industries in Nepal. The company has exported
50,000 readymade shirts to Turkey for equivalent NPR 10 millions and booked NPR 10 millions as income in their
account, however, the government of Turkey for the time being has suspended the equivalent NPR 5 millions amount to
be remitted to Sahara Garment P Ltd on the ground that the quota system as determined by the government of Turkey is
below than the goods as supplied by Sahara Garment from Nepal

b) Mr. Lakhan, Statutory Auditor of Radha Krishna Pvt. Ltd wants to verify cash on hand as on 31st Asadh, 2069. The
Management informs Mr. Lakhan that it is not possible to cooperate, as cashier has been out of station. Advise Mr.
Lakhan on how to deal with the situation.

c) A proprietary audit firm of Chartered Accountant has accepted the engagement to audit the accounts of a private
school with annual turnover of Rs 10 lakhs at the audit fee of Rs 8,500. The firm anticipates that the audit will consume
estimated time of 2 man days of the Chartered Accountant.

Answer:
a) NAS 07 Nepal accounting standards on revenue Para 18 has clearly defined the case of remit suspense. As per this
Para revenue is recognized only when it is probable that the economic benefits associated with the transaction will flow
to the entity. In some cases, this may not be probable until the consideration is received or until an uncertainty is
removed. For example, it may be uncertain that a foreign governmental authority will grant permission to remit the
consideration from a sale in a foreign country. When the permission is granted, the uncertainty is removed and revenue is
recognized. However, when an uncertainty arises about collectability of an amount already included in revenue, the
uncollectable amount or the amount in respect of which recovery has ceased to be probable is recognized as an expense,
rather than as an adjustment of the amount of revenue originally recognized.
Sahara Garment P Ltd has to adjust the balance suspended amount from the revenue which was recognized previously.
Otherwise the auditor should make qualification on the revenue reorganization.

b) The scope of audit may be limited for varied reasons, (i) the entity may impose restriction on scope of audit, (ii) the
limitation may be imposed by circumstances. When the audit is carried out under and as per statute, the auditor should
not accept the assignment when his duties are curtailed by agreement, unless required by any Law.

The Institute of Chartered Accountants of Nepal


3 of 62
Suggested Answers – Audit and Assurance
CAP II Examination – December 2012
When audit is carried out in accordance with the entity‟s terms voluntarily, the auditor may indicate his scope in his
audit report.
Sometimes, the circumstances may impose restrictions on audit scope. For example, if the auditor is appointed after the
year end, he may not be able to participate in inventory checking. Or sometimes, the records required may not be
available so that the auditor may not be able to check details in the manner he liked. Such limitations in scope may
warrant an auditor to express disclaimer of opinion or qualified opinion in his audit report depending upon the
circumstances.
The non co-operation of Radha Krishna Pvt. Limited will amount to limitation on scope of auditors.

c) As per directive issued by the council, a CA member holding COP shall charge the audit fee to his audit client and
the fee shall not be less than Rs 10,000. If the audit client is a financial institution regulated by Nepal Rastra Bank or
insurance company regulated by the Insurance Board, the audit fee shall not be less than Rs 40,000. However, above
minimum fee does not apply where the audit client is a Government Primary School/Community School with annual
turnover of less than Rs 5 lakhs and where the audit client is an organization with annual turnover of less than Rs 2
lakhs. In the present case the CA member has charged the audit fee of Rs 8,500 to a private school with annual turnover
of Rs 10 lakhs which is not covered by the exception for charging lower fee and hence the auditor should have charged at
least Rs 10,000 as audit fee. Hence the member seems to have not followed the directive of the council and accordingly
may be subject to disciplinary action.

4. Answer the following: (35=15)

a) How will you verify the retirement gratuity to employees?


b) An audit is not a guarantee that the financial statements are free from material misstatement, because absolute
assurance is not attainable. What are the factors which hinders the auditor to provide absolute assurance?
c) As statutory Auditor of XYZ Pvt. Ltd. you requested your client for sending letter for balance confirmations from
certain debtors, the client argue that since the said balances with debtors are under dispute and the matter is pending in
the court it is not necessary to ask balance confirmation.

Answer:
a) Verification of retirement gratuity
 Examine the basis on which payable to employees is worked out. The liability for gratuity may either be worked on
actuarial rules or agreement or the presumption that all employees retire on the balance sheet date.
 Verify computation of liability of gratuity on the aggregate basis.
 Check the amount of gratuity paid to employees who retired during the year with reference to number of years of
services rendered by them.
 See that the annual premium has been charged to profit and loss account in cash the concern has taken a policy from
the insurance companies.
 Ensure that the basis of computing gratuity is valid.
 Ensure that the accounting treatment is in accordance with NSA 24 Accounting and Reporting by Retirement
Benefits Plans'.

b) Factors hindering the auditor to provide absolute assurance: Auditor provides reasonable assurance on the
financial statements and is not in the position to provide absolute assurance due to the following factors:
i) Use of testing/sampling rather than 100% checking,
ii) The inherent limitations of internal control (for example, the possibility of management override or collusion),
iii) The fact that most audit evidence is persuasive rather than conclusive and
iv) The work undertaken by the auditor to form an audit opinion is permeated by judgment, in particular
regarding: (a) The gathering of audit evidence, for example, in deciding the nature, timing and extent of audit
procedures; and (b) The drawing of conclusions based on the audit evidence gathered, for example, assessing the
reasonableness of the estimates made by management in preparing the financial statements.

c) NSA 505, “External Confirmations”, establishes standards on the debtor‟s use of external confirmation as a means
of obtaining audit evidence. It requires that the auditor should employ external confirmation procedures in consultation
with the management. The auditor may come across certain situations in which the management may request him not to
seek external confirmation from certain parties because of dispute with the debtors, etc. The management, for example,
might make such a request on the grounds that due to a dispute with the particular debtor, the request for confirmation
might aggravate the sensitive negotiations between the entity and the debtor. In such cases, when an auditor agrees to
management‟s request not to seek external confirmation regarding a particular debtor, the auditor should consider
validity of grounds for such a request and assess management‟s integrity and obtain evidence to support the same. The
auditor should also ask the management to submit its request in a written form, dealing therein the reasons for such a
request. The auditor agrees to management‟s request not to seek external confirmation regarding a particular matter, the
auditor should document the reasons for accepting to the management‟s request and should apply alternative procedures
to obtain sufficient appropriate evidence regarding that matter. While considering the validity of request, in case the
auditor reaches at a conclusion that the same was not valid, he may appropriately modify the report.

5. Comment on the following situations/statements. (35=15)


The Institute of Chartered Accountants of Nepal
4 of 62
Suggested Answers – Audit and Assurance
CAP II Examination – December 2012

a) Though the audit for the year 2068/69 was completed, but not signed the financial statements of X Limited by the
board of directors. In the mean time, the board has decided to stop the operation of Z Limited, one of the major
subsidiary companies of X Limited with effect from Marg 1, 2069.

b) What is the responsibility of the professional accountant " When he knows a material error or omission in a tax
return of a prior year (with which the professional accountant may or may not have been associated), or of the failure to
file a required tax return?

c) Ramesh Sharma was Chief Finance Officer of A Ltd for the period 1 Srawan 2064 to 15 Ashad, 2069. He joined
XYZ Chartered Accountant firm from 1 Srawan 2069 as Senior Audit Manager. A Ltd. has approached the firm on 15
Shrawan 2069 for the audit of accounts for the year 2068/69 with Ramesh Sharma as Engagement Manager of the audit
team. Please provide your opinion on what the audit firm should do?

Answer:
a) NAS 05 section 21 has defined the non – adjusting events after the balance sheet date, but it should be disclosed in
the balance sheet. The sections this events as follows:
If non-adjusting events after the balance sheet date are material, nondisclosure could influence the economic decisions of
users taken on the basis of the financial statements. Accordingly, an entity shall disclose the for each material category of
non-adjusting event after the balance sheet date:
(a) the nature of the event; and
(b) an estimate of its financial effect, or a statement that such an estimate cannot be made.

Section 22 has defined the examples of non adjusting events after the balance sheet date are as follows:
(a) a major business combination after the balance sheet date (NAS 21 Business Combinations requires specific
disclosures in such cases) or disposing of a major subsidiary;
(b) announcing a plan to discontinue an operation;
(c) major purchases of assets, classification of assets as held for sale, other disposal of assets, or expropriation of major
assets by government;
(d) the destruction of a major production plant by a fire after the balance sheet date;
(e) announcing, or commencing the implementation of, a major restructuring;
(f) major ordinary share transactions and potential ordinary share transactions after the balance sheet date (NAS 26
Earnings Per Share requires an entity to disclose a description of such transactions, other than when such transactions
involve capitalization or bonus issues, share splits or reverse share splits all of which are required to be adjusted under
NAS 26);
(g) abnormally large changes after the balance sheet date in asset prices or foreign exchange rates;
(h) changes in tax rates or tax laws enacted or announced after the balance sheet date that have a significant effect on
current and deferred tax assets and liabilities (see NAS 09 Income Taxes);
(i) entering into significant commitments or contingent liabilities, for example, by issuing significant guarantees; and
(j) commencing major litigation arising solely out of events that occurred after the balance sheet date.
In the financial statements of X Limited, a proper disclosure is required regarding the closure of the subsidiary company
Z Limited and its effects though the financial figures will not be changed in the balance sheet.

b) As per Section 5.8 of the Code of Ethics of the institute of the chartered accountants of Nepal in such case
professional accountant is responsible for:

i. Promptly advise the client or employer of the error or omission and recommend that disclosure be made to the
revenue authorities. The professional accountant is not obligated to inform the revenue authorities.

ii. If the client or the employer does not correct the error the professional accountant:

(a) should inform the client or the employer that it is not possible to act for them in connection with the return or other
related information submitted to the authorities; and

(b) should consider whether continued association with the client or employer in any capacity is consistent with
professional responsibilities.

iii. If the professional accountant concludes that a professional relationship with the client or employer can be
continued, all reasonable steps should be taken to ensure that the error is not repeated in subsequent tax returns.

c) Ramesh Sharma was the Chief Finance Officer of A Ltd. for almost entire year 2068/69. He has left A Ltd just a
month before the audit firm was approached by A Ltd for audit of 2068/69 and proposes his name to be engaged as
engagement manager in the audit team. This situation may result into self-interest, self review and familiarity threat to
the firm. Hence the firm should request A Ltd. that the firm cannot assign Ramesh as an audit team member. If A Ltd
does not accept the audit team without Ramesh as senior member in the audit team from the firm, the audit firm should
not accept this engagement.
The Institute of Chartered Accountants of Nepal
5 of 62
Suggested Answers – Audit and Assurance
CAP II Examination – December 2012

6. Write short notes on the following: (42.5=10)


a) Legal liabilities of auditor
b) Internal Audit
c) Engagement Letter
d) "During payments vouching, the auditor does not merely check proof that money has been paid away".

Answer:
a) Legal Liabilities of auditor:
Auditors must always perform their work complying the basic principles of audit, which are, integrity, objectivity,
independence, confidentiality, professional competency, due care and technical standards.

If the same is not complied with, then they can be held liable on account of:
i) Non compliance with Nepal Standards on Auditing; and
ii) Failure to protect the interest of stakeholders relying upon the audited financial statements;

Accordingly, the auditors‟ liability falls under three categories:


i) To their clients (company itself);
ii) To third parties in case of negligence; and
iii) Civil and criminal liabilities.

b) Internal Audit: Internal audit is an independent, objective assurance and consulting activity designed to add value
and improve an organization's operations. It helps an organization accomplish its objectives by bringing a systematic,
disciplined approach to evaluate and improve the effectiveness of risk management, control, and governance processes.

c) Engagement Letter: It is letter issued by the auditor to the auditee which includes written terms of engagement.
This letter basically sets out the responsibilities of the auditor and the auditee and includes other information such as
audit fee and its payment terms, out of pocket expenses etc. This letter helps to resolve the confusion which may arise
during or subsequent to audit.

d) Vouching is a substantive audit procedure which aims at verifying the genuineness and validity of a transaction
contained in the accounting records. It involves examination of documentary evidence to support the genuineness of
transaction.
Thus the object of vouching the payments of a business is not merely to ascertain that money has been paid away; but the
auditor aims to obtain reasonable assurance in respect of following assertions in regard to transactions recorded in the
books of account that :
i) a transaction is recorded in the proper account and revenue or expense is properly allocated to the accounting period;
ii) a transaction pertains to entity and took place during the relevant period;
iii) all transactions which have actually occurred have been recorded;
iv) all transactions were properly authorized; and
v) transactions have been classified and disclosed in accordance with recognizedaccounting policies and practices.
Thus, it is through vouching that the auditor comes to know the genuineness of transactions recorded in the client‟s
books of account wherefrom the financial statements are drawn up.
Thus, the auditor‟s basic duty is to examine the accounts, not merely to see its arithmetical accuracy but also to see its
substantial accuracy and then to make a report thereon.

7. Distinguish between (25=10)


a) Statutory & System Audit
b) Peer review, hot review and cold review

Answer:
a) The major differences between statutory and system auditing can be described as follows:
(i) Purpose: The statutory auditing is basically concerned with the opinion that whether the historical information
recorded is correct or not, whereas the system auditing emphasizes on effectiveness and efficiency of operations for
future performance.
(ii) Area: Statutory audits are restricted to the matters directly affecting the appropriateness of the presented financial
statements whereas the system audit covers all the activities that are related to efficiency and effectiveness of operations
directed towards accomplishment of objectives of organization.
(iii) Reporting: The statutory audit report is sent to all stock holders, bankers and other persons having interest in the
organization. However, the system audit report is primarily for the management.
(iv) End task: The statutory audit has reporting the findings to the persons getting the repost as its end objectives,
however, the system audit is not limited to the reporting only, but includes suggestions for improvements also.
(v) Auditor: In Statutory Audit generally opinion on the financial statement has to be expressed by COP holder of
ICAN, though system audit could be done in-house or by outsourcing where it is not necessary for carrying out by COP
holder.

b) Peer Review
The Institute of Chartered Accountants of Nepal
6 of 62
Suggested Answers – Audit and Assurance
CAP II Examination – December 2012
This is a critical independent review of one public accounting firm practices by another public accounting firm. It is a
review of the firm‟s accounting and auditing practices. It is intended that the review be done by practitioners upon fellow
practitioners.
Such an external review offers a more objective evaluation of the quality of performance than could be done by self
review.
Peer review study the adequacy of the firms established quality control policies and tests to determine the extent of the
firms compliance to these policies.
Suggestions for improvement to the system are outlined in a letter of comments issued by peer reviewers to the reviewed
firm. If a firm fails to take appropriate corrective action, various actions may be imposed e.g. suspension from
membership.
In carrying out the review it is limited to:-
 • Professional aspects of the practice.
 • The overall total quality control policies.
 • Professional aspect of the firms accounting and auditing practices like maintenance of working papers and work
products such as report to the financial statements.

Hot Review
Is an independent review of an audit by a suitably qualified firm of auditors before the issue of the audit report? The
review includes evaluation of the engagement, working papers, reports and the firm compliance with established quality
control policies and professional standards.
Hot review helps the reviewed firm to make necessary changes on their report or have confidence that what they are
reporting was well conducted and no liability can result from their report.
Hot reviewers assess how the firm accepted their engagement i.e. whether they followed professional ethics and whether
they were qualified to act as auditors for the entity. Secondly they review the firms audit plans and audit programs to
consider how duties were assigned delegated and directed. They address the critical areas by evaluating the working
papers and how the audit evidence was gathered i.e. audit tests and procedures were well performed according to the
established standards.
Finally the reviewers will evaluate the report and make conclusion in writing as to whether all the matters raised have
been satisfactorily satisfied/resolved by the firm. They will also recommend whether further tests are necessary to come
up with comprehensive reports.

Cold Review
It is the involvement of an independent accounting firm or a partner to provide assurance that all the firms‟ in house
quality control policies have been complied with as well as provide a second opinion that the audit was performed in
accordance with generally accepted audit standards.
Cold review is an objective examination of an audit assignment after its completion referred to as post audit review.
Cold review assess the policies and procedures the firm used to conduct its audit. This involves audit planning and
ascertaining that the audit team followed the audit plans together with audit programs. It also ascertains that work was
well documented through working papers and judgment or conclusion made.
Cold review report to staff adherence to ISAS and review work done by each staff and see that it is according to the
policies of the firm.

The Institute of Chartered Accountants of Nepal


7 of 62
The Institute of Chartered Accountants of Nepal
Suggested Answers of Corporate and Other Laws

CAP II Examination- December 2012

Part "A"
1. Answer the following questions:
a) M/s. Simon Ltd. was wound up by the court. The official liquidator invited claims from its creditors which stood as
under:
Income Tax dues 11 lakhs
Sales Tax dues 5 lakhs
Dues of workers 25 lakhs
Unsecured loans Payable to directors 25 lakhs
Trade creditors who supplied raw material 15 lakhs
Secured creditors being the banker of the Company 75 lakhs
Total Rs. 156 lakhs

Official liquidator could realize Rs. 80 lakhs by sale of assets and realization made from the Company debtors which is not
sufficient to pay to all the creditors. Decide the order of priority for payment to creditors explaining the relevant provisions
of the Companies Act, 1956. 8

b) A, signs as maker a promissory note and gives it to B who delivers it to C as a gift. C, with a view of cancelling the
note makes two pieces and throws it to a container. D, who was in service of cleaning roads, finds the pieces of note and
fixes them as there was no sign of tear. He passes the note to M who has received in good faith for consideration. Can M
recover the amount stated therein from A? Decide with reasons. 6

c) Mr. Hari Singh was appointed as a managing director of the plaintiff company - Reliance Motors India Co. on the
condition that he shall not at any time while he shall hold the office of a managing director or afterwards, solicit or entice
away the customers of the company. Shortly afterwards, his employment was terminated under an agreement and he
opened a business in the name of a separate company Golden Gate Automobiles Co. and began solicit the customers of the
Reliance Motors India Co. When the case filed by the Reliance Motors India Co., he argued that the business is of the
Golden Gate Automobiles Co. Advise Reliance Motors India Co. whether their claim is maintainable against Hari Singh?
Decide with relevant provisions and principles enshrined under the Companies Act, 1956. 8

d) Explain as to why the combination of 'not negotiable' with 'Account Payee' crossing is considered the safest form of
crossing a cheque. 3

Answer:
a)
Where the assets of the company available for payment of general creditors are insufficient to meet their claims, certain
unsecured debts are paid before the debenture holders. Sec 530 says there shall be paid in priority to all other debts.
1. Rates and taxes having become due to central or state govt.
2. All wages or salary at an employee
3. Compensation payable under the workmen's compensation in respect of death or disablement of any officer or employee
4. All sum due to any employee from a provident, pension, gratuity
5. The expenses of any investigation held under S. 235 or S.237
6. All accrued holyday remuneration payable to any employee
7. All amount due in respect of contributions payable during 12 months
Under Section 529, in the winding up of an insolvent company, the rules shall prevail and be observed with regard to (a)
debts provable; (b) the valuation of annuities and future and continent liabilities; and (c) the respective rights of secured
and unsecured creditors; as are in force for the time being under the law of insolvency with respect to the estates of persons
adjudged insolvent provided that the security of every secured creditor shall be deemed to be subject to a pari passu charge
in favour of the workmen to the extent of the workmen's portion therein and where a secured creditor instead of
relinquishing his security his security and proving his debt, opts to realize his security.
(a) the liquidator shall be entitled to represent the workmen and enforce such charge ;
(b) any amount realised by the liquidator by way of enforcement of such charge shall be applied
rateably for the discharge of workmen's dues ; and (c) so much of the debt due to such secured creditor
as could not be realised by him by virtue of the foregoing provisions of this proviso or the amount of
the workmen's portion in his security, whichever is less, shall rank pari passu with the workmen's dues
for the purposes of section 529A.
Suggested Answers – Corporate and Other Laws
CAP II Examination – December 2012

All persons who in any such case would be entitled to prove for and receive dividends out of the assets
of the company, may come in under the winding up, and make such claims against the company as
they respectively are entitled to make by virtue of this section:
Provided that if a secured creditor instead of relinquishing his security and proving for his debt
proceeds to realise his security, he shall be liable to pay his portion of the expenses incurred by the
liquidator (including a provisional liquidator, if any) for the preservation of the security before its
realisation by the secured creditor.
Section 529A has given the power of the overriding preferential payments. Under Section 529A of the Companies Act,
1956, (i) workmen's dues, and (ii) debts due to secured creditor shall be paid in priority of all debts and shall be paid in full,
unless the assets are insufficient to meet them, in which case they shall abate in equal proportions.

Income tax dues and sale tax dues are preferential creditors under section 530 of the Act, and subject
to the provisions of section 529A, the same may be paid in priority to the claims of unsecured
creditors.

In the present case, the available funds are only to the extent of Rs. 8m which will be distributed
amongst the secured creditors and workmen in proportion to their dues, as follows:
Workmen (1/4th of Rs. 8m) Rs. 2m
Secured creditors (3/4th of Rs. 8m) Rs. 6m

As such, the dues of preferential creditors (namely, Income tax and sales tax dues) and unsecured
creditors (unsecured loan and trade debtors) cannot be paid any amount.
a) A person who receives negotiable instrument for consideration, before its maturity and in good faith without having
notice of defective title is known as the holder in due course. Certain defenses which can set up as against a holder cannot
set up as against the holder in due course. The holder in due course can get good title in the instrument though he has
received it from the transferor whose tile in the instrument is defective.

In this case, M is the holder in due course as he had received the promissory note without having knowledge of its
cancellation as it could not be found whether it was tore or not. Further, as M had received the Note for consideration, no
claim can be made against him as the instrument was theft and delivered to C by way of gift. Therefore, A cannot refuse to
pay the amount stated therein to M. He can recover the amount as mentioned in the note.

b) The separate legal existence of a company cannot be used as a means to achieve some illegal or fraudulent purposes.
The courts will refuse to uphold the separate existence of the company where it is formed to defeat or circumvent law, to
defraud creditors or to avoid legal obligations. This principle was laid down by the court in the case of Gilford Motor Co
vs. Horne (1930)1Ch 935.

In this case the company Golden Gate Automobiles Co. incorporated by Mr. Hari Singh is a mere cloak or sham for the
purpose of enabling him to commit a breach of his covenant against solicitation. The defendant company was just a
channel used by Mr. Hari Singh for the purpose of enabling him to obtain the advantage of the customers of the plaintiff
company, and the defendant company ought to be restrained as well as the defendant Hari Singh. Therefore, their claim is
maintainable against Hari Singh.

c) The addition of the words 'not negotiable' in a crossed cheque does not restrict the transferability of the instrument, but
the cheque is deprived of its special feature of negotiability. The general rule about the negotiability is that the holder in
due course of a bill or promissory note or cheque takes the instrument free from any defect which might be existing in the
title of the transferor. According to Section 130 of the Negotiable Instruments Act, 1881 a person who takes a cheque
bearing 'not negotiable' marking shall not have, and shall not be capable of giving, a better title to the cheque which the
person from whom he took it, had. A bank therefore should be extra careful in paying such cheques. The payment should
be made only after he is satisfied that the person demanding payment is the person entitled to receive it.

Account payee crossing directs the collecting banker to collect it for the payee only and warns that if the amount is
collected for some else, he may be held liable for damages.

In view of the advantages explained above, the combination of 'not negotiable' and 'A/C payee' crossing can be considered
the safest form of crossing.
The Institute of Chartered Accountants of Nepal
9 of 62
Suggested Answers – Corporate and Other Laws
CAP II Examination – December 2012

Part "B"
2. Answer the following questions:
a) D Ltd. was incorporated in accordance with the Company Act, 2063 (2006). It has 100 shareholders. The Articles of
Association (AOA) provides the quorum for the proceedings of the meeting of the company at least 10 shareholders of the
total shareholders representing more than 60 percent of the total numbers of allotted shares of that company present either
in person or by proxy. Further, it also provides that the matters relating to increase the authorized capital of the company
should be presented in general meeting of the company as a special resolution. It has got its license to commence its
business in February 1, 2011. As the company law and its rules, company intended to increase its authorized capital.
However, annual general meeting of the company could not hold except first annual general meeting even the pressure of
the shareholders. It is said to be that the shareholders of the company are the owner of the company. So, in relation to the
Companies Act, 2063 (2006) solve following problems with the help of the facts or the legal provisions of the Companies
Act. (5+3=8)
i) How can the shareholders compel to the company to hold annual general meeting? Explain the legal provisions.

ii)The Company called annual general meeting due to the pressure of the company's shareholders and passed resolution to
increase authorized capital having presence 6 in numbers that represents only 40% shareholders of the total numbers of
allotted share of that company. Is the resolution passed is valid or any else?

b) What do you understand by breach of contract? Explain briefly about the remedy available for the breach of contract.
7

Answer:
a) Shareholders of the Company have certain rights and liability only to the maximum value of shares which may have
subscribe or undertaken to subscribe.

D Ltd. could not hold the annual general meeting. In this connection, section 76 of the Companies
Act, 2063 (2007) has provided about the procedure of the calling annual general meeting. This section
provides alternative way if any public company could not manage to hold annual general meeting.

1) Directions of Company Registrar Office:


Every public company shall hold its annual general meeting every year within six months after the
expiry of its financial year. If such annual general meeting could not be held within six months after
the expiry of its financial year, the company registrar office may give direction to call the annual
general meeting of such company.

2) Making a petition by any shareholder:


Even the company fails to call the annual general meeting within three months after the receipt of the direction as referred
to above any shareholder may make a petition for, setting out the matter, to the court. Where such petition is made, the
court may either cause to hold the annual general meeting or issue any other appropriate order.
According to the above legal provisions, if any company couldn't hold annual general meeting within
time mentioned above the company registrar office may give direction to call the general meeting.
Any shareholder may make a petition setting out the matter to the court if the company unable to hold
the annual general meeting within three months from the date of receipts of the direction.

Here, the second question is testing the validity of increasing authorized capital. This resolution is
passed by the shareholders meetings with the presence of 6 members that represents only 40%
shareholders of the total numbers of allotted share of that company. This quorum is less than the
provision of the companies' AOA. In another side, section 73(2) of the Companies Act, 2063 (2007)
has given right to state required quorum of large number. Unless the article of association of a public
company provides for a larger number for the quorum, the proceeding of the meeting of the public
company shall be conducted unless at least three shareholders of the total shareholders, representing
more than fifty percent of the total number of allotted share of that company are present either in
person or by proxy." However, company is facing an extraordinary situation that is the resolution of
increasing authorized capital could not pass. So, the provision provided in the section 76(4) is the way-
The Institute of Chartered Accountants of Nepal
10 of 62
Suggested Answers – Corporate and Other Laws
CAP II Examination – December 2012

out of solving the extraordinary situation. It has provided circumstantial quorum despite the provision
of AoA or in the section 73(2). The shareholders present in the general meeting called pursuant to the
order of the court under Sub-section (3) shall be deemed to be a quorum. Hence, it is clear that the
resolution increasing authorized capital with fewer quorums is valid.
However such exemption has not been provided in case of meeting called as per the direction of the
Company Registrar Office.
b) Section 82 of the Contract Act, 2056 has defined the condition of the breach of contract. As per the section:
(1) In case any party to a contract does not meet liability under the contract, or gives a notice to the other party that he/she
will not perform the work to be performed under the contract, or in case his/her action or conduct shows that he/she is
incapable of performing the work under the contract, he shall be deemed to have breached the contract. (2) In case a party
has broken the contract under Sub-section (1), or in case his/her action or conduct shows that he/she has not basically
compiled with the contract, the other party shall not be compelled to perform the contract, and may cancel the contract by
furnishing a notice thereof to the other party.

In case of breach of contract, the aggrieved party has becomes entitled to get the following legal remedies;
 Suit for Rescission of cancellation of contract- Rescission means setting aside or cancelling the contract. In other
words, the term “Rescission” may be defined as the cancellation of the contract. When a contract is broken by one party,
the other party may treat the breach as discharge and refused to perform his part of the contract, i.e. putting an end to the
contract. On the rescission of the contract, the aggrieved party is discharged from all the obligations under the contract.
 Suit for Damages; The Damages are a monetary compensation allowed to the injured party for the loss or injury
suffered by him as a result of the breach of contract. In the event of breach of contract, the other party earns certain rights
including the right to claim damages or loss arising there from. The object of awarding is to put the aggrieved party in the
same position, has the contract been performed. The term „damages‟ is used to mean compensation in money as a
substitute for the promised performance. The fundamental principle underlying damages is not punishment but
compensation. Damages are to be awarded for losses which naturally arose from the breach of contract. The law of contract
does not seek to punish the guilty. The guilty party is liable to pay damages to the aggrieved party. The court will compel
the party in breach to make good the loss by paying to the other party.
 Suit for Quantum Meruit; The Latine phrase “Quantum Meruit” means payment in proportion to the amount of “work
done” or “reasonable value of work done”. A person can under circumstances, claim payment for work done of goods
supplied without any contract and in cases where the original contract has terminated by breach of contract by one party or
has become void for some reasons. This is known as “Doctrine of Quantum Meriut”. This doctrine is applies where there is
no express promise to definite remuneration to a person.
 Suit for specific performance; Specific performance means the actual carrying out of the contract as agreed. Where
damages are not an adequate remedy for breach of the contract, the court may direct the party in breach to carryout his
promise according to the terms of the contract. This is called “specific performance “of the contract. Specific performance
of the contract cannot be claimed as a matter of right .following are the circumstances , the claim for specific performance
is not applicable( Section 86(2) ) of the Nepalese contract Act, 2056;
i) Where compensation for the breach is adequate.
ii)Where the court cannot supervise the execution from time to time.
iii) Where the contract is of personal nature s(personal skill, efficiency, or Knowledge).
iv) Where a specific performance is not applicable due to the change of circumstances,
v) Where the breacher party itself has claimed for a specific performance against the injured party.
 Suit for injunction; Injunction means „ to stop doing something”. Injunction is also one of the remedies for an injured
party . Injunction is a court order that restraints the breacher party from doing wrong or continuing the wrongful act,
complained. Such order is usually granted by the court, to enforce negative stipulations in the case where the damage can
not be an adequate relief for the injury party. Such remedy is appropriate where there is an anticipatory breach of contract.
It is a kind of preventive relief for the aggrieved party.

3. Answer the following questions: (2×5=10)


a) Discuss how a disciplinary action is taken against the members and what punishment can be imposed to them by
Council.
b) Discuss the legal provisions for settlement of disputes of bonus under Bonus Act, 2030.

Answer:
a) Under section 14 (1) of the Act, if a member has acted as against the provisions of the Act or rules or code of conduct
on the application of anyone or such is known to the ICAN, to investigate upon the allegation and to recommend for the
necessary action to the council a Disciplinary committee shall be formed by council. Similarly under section 14 (5) of the
Act has provided on the necessary investigation conducted by the Disciplinary committee if a member is found guilty the

The Institute of Chartered Accountants of Nepal


11 of 62
Suggested Answers – Corporate and Other Laws
CAP II Examination – December 2012

committee with its decision may recommend the council for the necessary action. On the recommendation of the
committee the council may punish the accused members as follows:-
(a) Reprimanding;
(b) Removing from the membership for a period not exceeding Five years;
(c) Prohibiting from carrying on the accountancy for any specific period;
(d) Canceling the professional certificate or membership.
Under section 14 (1) it makes mandatory that a reasonable opportunity must be given of being heard before imposing the
punishment and the member punished if not satisfied with the decision of the council may apply to the court of Appeal to
challenge the decision of the council.

b) The Bonus Act, 2030 assures the bonus to the employees out of the profits earned by a factory or enterprise. The Act
also has made legal provisions for the settlement of disputes which may rise between the employee and management.
Section 16 (1) provides that if any dispute arises between employee and management with respect to the bonus, the Labour
Office shall resolve such dispute by negotiations having invited both the parties. Similarly, if the dispute could not be
resolved by negotiation the Labour Office shall ask to the concern enterprise and employees to produce necessary
documents and statements of accounts and shall give a decision on the basis of such documents and statements. When the
decision is given by the Labour Office, the party who is dissatisfied with the decision may appeal to the Labour Court
within 35 day of receipt of such notice and the decision made by the Labour Court shall be final for the matter.

4. Answer the following questions: (2×5=10)


a) When does a public limited company converse into a private limited company? Explain the legal provisions.
b) What are the terms to be abided by any company incorporated under the companies Act, 2063 in addition to those set
forth in the Companies Act, 2063, Memorandum of Association or Articles of Association?

Answer:
a) Section, 14 of the Companies Act, 2063 under different clauses has stated the provisions for the conversion of a public
company into a private company. Sub-section 1 of section 14 reads thus:
In the following circumstance, a public company shall be converted into a private company under this section:
-If the number of shareholders of the public company becomes less than seven,
-If the public company fails to maintain its paid-up capital under section 11 or the paid-up capital as
referred to in section 11 is not maintained because of reduction in capital pursuant to section 57.
Provided, however, that this provision shall not apply to the company as referred to in sub-section (2)
of section 11.
In the event of occurrence of a circumstance as referred to in sub-section (1), the concerned public company shall make
necessary amendments to its memorandum and articles of association and convert it into a private company within six
months. After making the necessary amendments in the memorandum and articles of association, the company, within 30
days after the making of such amendment, shall make an application to the Office of the Company Registrar with the
prescribed fees for being converted into a private company. The Office on the receipt of an application shall mention in the
company register the contents of conversion of such company into a private company and give a company conversion
certificate within sixty days.

b) A company incorporated under this Act shall abide by the following terms, in addition to those set forth in this Act,
memorandum of association or articles of association: Section 10
(a)The company shall carry on all of its activities and transactions by its name.
(b) A private company shall add the words “private limited‟‟ to its name as the last words and a public company shall add
the word “limited” to its name as the last word. However, this provision shall not apply to a company not distributing
profit.
(c) A private company shall not sell its shares and debentures publicly.
(d) A private company shall not pledge, or otherwise transfer title to, its securities to any person other than its shareholder
without fulfilling the procedures contained in the memorandum of consensus agreement,
(e) A company shall not open a partnership or private firm.
(f) Except as otherwise provided in this Act, a company not distributing profits shall not distribute dividends among its
members or pay, directly or indirectly, any amount to a member or his/her close relative.

5. Answer the following questions: (3×5=15)


a) What are the acts prohibited and punishment prescribed in Insurance Act, 2049?
b) Define capital fund under BAFIA, 2063.
c) Highlight the incentives for the foreign investors assured by the Foreign Investment and Technology Transfer Act,
2049
The Institute of Chartered Accountants of Nepal
12 of 62
Suggested Answers – Corporate and Other Laws
CAP II Examination – December 2012

Answer:
a) Insurance Act, 2049 has provided in its section 36 about the acts prohibited or punishment thereto as follows:

i) Violation of Law:
 If any insurer or the Director of the Insurer, employee or Surveyor, Broker or Insurance Agent knowingly violates this
act or the rules made under this act or order or directives;
 If such authority does not perform any function to be performed or does any act not to be done
 The Insurance Board may punish to such Insurer or the Director, employee or *Surveyor, Broker or Insurance Agent
with a fine ranging from three thousand rupees to ten thousand rupees. If such offense has been made frequently, he will be
fined at the rate of extra five hundred rupees for each subsequent offense.

ii) Non-Compliance of Procedures:


 If any Insurer or Insurance Agent or Broker has made any Insurance Business without following the procedures to be
followed pursuant to this Act it shall be deemed to be the offence.
 The Security Board may fine such authority up to ten thousand rupees.

iii) Improper activities or malafide intention:


 The accounts, records, register, details, information or any other documents should be maintained, prepared, formed or
submitted pursuant to this Act or the Rules made under this Act. If the job has not done in time by knowingly or with
malafide intention he/she should be liable for his mal performance.
 Should not maintained or submitted the false details or documents by any-body;
 If there is beyond performance he/she is liable of punishment with a fine up to thirty thousand rupees or imprisonment up
to two years or with both.

b) Section 2 (r ) of the BAFIA Act, 2063 has defined the terms Capital Fund. As per this section “capital fund”
means the total of the primary and supplementary capital of a bank or financial institution, and this term includes any other
fund of the institution as prescribed as such by the Nepal Rastra Bank from time to time.

Section 42 of the Act has explained the capital fund for the licensed institution.
(1) Every licensed institution shall maintain a capital fund in the ratio prescribed by the Rastra Bank on the basis of its total
assets or total risk-weighted assets.
(2) If any bank or financial institution fails to maintain the capital fund as referred to in Sub-section (1), the Board shall
give information thereof to the Rastra Bank within thirty five days. The information so given shall also be accompanied by,
inter alia, the reasons for the failure to maintain the capital fund and the plan or program prepared by the Board to increase
the capital fund and restore it to its previous condition.
(3) On receipt of the information referred to in Sub-section (2), if the Rastra Bank considers the plan or program submitted
by the Board to be reasonable, it may give a directive to the concerned bank or financial institution to implement such plan
or program; and if any amendment or alteration is to be made in the proposed plan or program, it may give a directive,
accompanied by the reasons for such amendment or alteration, to the concerned bank or financial institution to amend or
alter such plan or program and implement the same.
(4) If any bank or financial institution fails to meet the obligation referred to in this Section or if it does not appear that it
can do so immediately, the Rastra Bank may take action referred to in Section 37 against such bank or financial institution.
(5) Notwithstanding anything contained elsewhere in this Section, if it is found that the concerned bank or financial
institution has failed to maintain its capital fund as provided for in Sub-section (1) by the reason of the failure of the Board
of Directors to comply with the directives given by the Rastra Bank from time to time or by the reason of malice
recklessness or mala fide intention of the director or chief executive, the Rastra Bank may punish such director or chief
executive with a fine equal to the amount involved; and if such act of the director or chief executive is found to have
caused any loss or damage as a result of the failure to maintain the capital fund, the Rastra Bank, the concerned bank or
financial institution or any shareholder holding up to five per cent of the paid-up capital of such bank or financial
institution may either individually or collectively file a petition with the court against such director or chief executive for
the recovery of that amount, in accordance with laws in force.

c) Section 2(1) of the Foreign Investment and Technology Transfer Act, 2049 has given the definition of the term
foreign investment. It defines- the term foreign investment means the following investment made by any foreign investor in
any industry of Nepal:
i) Investment in share capital.
The Institute of Chartered Accountants of Nepal
13 of 62
Suggested Answers – Corporate and Other Laws
CAP II Examination – December 2012

ii) Re-investment of income received from the investment in share capital.


iii) Investment as a loan or loan facilities.
There are numbers of facilities given by the Act to attract the foreign investment in Nepal. Pursuant to section 5 of the Act
the following facilities are provided to the foreign investors:
i) Facilities to take away the foreign currencies from Nepal:
The foreign investors can take away the following amount in the foreign currencies from Nepal:
1.Amount received from sale of whole or some part of share.
2.Amount received as profit or dividend of foreign investment.
3.Amount received as interest of loan or the payment of interest.
4.Amount received under the technology transfer agreement.

ii) Visa facilities: Under section 6 of the Act there are three types of visa issued to the foreign
investors. They are- Non-tourist visa, Business visa and Residential visa.
Foreign Investment and Technology Transfer Act 2049 (FITTA 2049) has different provisions on visa
facilities. As per FITTA 2049, a foreign national visiting Nepal in connection with undertaking any
study or carrying out any research with the objective of making investment in Nepal shall be provided
a non tourist visa for up to six months.
A foreign investor or dependent family or authorized representative of a foreign investor and department family of such
authorized representative shall for the purpose of stay in Nepal be provided a business visa until the foreign investment is
retained.
A foreign investor who, at a time, makes investment in an amount no less than one hundred thousand United States dollar
or in convertible foreign currency equivalent thereto, and his/her dependent family shall be granted a residential visa until
such investment is retained.
6. Answer the following questions: (2×5=10)
a) What is insider trading? Who can obtain inside information? Is there any provisions regarding the punishment about
the conviction of offense of insider trading?
b) Professional Solution Ltd announced 5th annual general meeting of the company in Kantipur daily published on 23rd
Katrik 2069.There is no mention any date, time duration, place and agenda of the meeting. Company published a notice for
presence of shareholders in annual general meeting only. Is this meeting valid under Company Act, 2063? Is there any
provision regarding the process of calling annual general meeting of public company? Explain.

Answer:
a) Section 91 of Securities Act, 2063 has defined insider trading as follows:
(1) If any person deals in securities or causes any other person to deal in securities on the basis of any
insider information or notice that are unpublished or communicates any information or notice known
to such a person in the course of the discharge of his or her duties in manner likely to affect the price
of securities such a person shall be deemed to have been committed an insider trading in securities.
(2) Notwithstanding anything contained in Sub-section (1), any transactions already carried on shall
not be deemed to be affected at all merely by the reason that an insider trading has been committed.
Section 92 of the Act has defined the person who is likely to be involved in insider trading: The persons shall be deemed to
be those who have access to the insider information or notice not published by any body corporate:

(a) A director, employee or a person, who can obtain any information or a notice in the capacity of a shareholder of that
body corporate,

(b) A person who can obtain any information or a notice in the capacity of a professional service provider to that body
corporate,

(c) A person who can obtain any information or a notice having a direct or indirect contact with the person or source as
specified in Clauses (a) and (b)

Punishment: A person who commits an insider trading as referred to in Section 91 section 91 shall, upon being convicted
of the offense of insider trading, be liable to the punishment with a fine equal to the amount in controversy or with
imprisonment for a term not exceeding one year or with both punishments.

The Institute of Chartered Accountants of Nepal


14 of 62
Suggested Answers – Corporate and Other Laws
CAP II Examination – December 2012

b) Professional solution ltd company announced a general meeting by publishing a notice in Kantipur Daily newspaper.
In this notice there is no any information regarding the business of the meeting. According to companies Act, 2063, section
67 (2) A public company shall a notice specifying the place, date, and agenda of meeting to every shareholder at the
address supplied by that shareholder to the company, in advance of at least 21 days to hold the annual general meeting A
notice thereof shall also be published at least twice in a national newspaper. Section 67 (3) No decision shall be taken in
any general meeting on any matter which has not been notified in advance pursuant to sub-section (2) except in the
following circumstances;
 Except as otherwise provided in the other sections of this Act, if the shareholders representing sixty seven percent of the
total shares of the company who rare entitled to vote at the general meeting attend in person or by proxy and vote in favour
of taking decision on any matters.
 If the matter was already notified for being transacted in any general meeting which has been adjourned.
Company Act, 2063 make compulsion to call annual general meeting before 21 days with mentioning the information
about the date, time, place and agenda for the meeting that must be send to every shareholders of the company. This is
mandatory for the company management. This is not optional. Here the professional solution limited does not fulfill the
criteria for calling the annual general meeting of the company according to companies Act, 2063. So the validity of the
meeting is not there and that meeting should not be accepted by the company Registrar office.

7. Write short notes on the followings: (3×5=15)


a) Objectives of Nepal Rastra Bank
b) i) Promissory Note ii) Bills of Exchange
c) Propriety Audit

Answer:
a) Objectives of Nepal Rastra Bank
Section 4 of Nepal Rastra Bank Act, 2058 has defined the objectives of the bank as follows:
(a) To formulate necessary monetary and foreign exchange policies in order to maintain the stability of
price and balance of payment for sustainable development of economy, and manage it;
(b) To promote stability and liquidity required in banking and financial sector;
(c) To develop a secure, healthy and efficient system of payment;
(d) To regulate, inspect, supervise and monitor the banking and financial system; and
(e) To promote entire banking and financial system of the Nepal and to enhance its public credibility.

The Bank shall, without any prejudice to the objectives referred to above, extend co-operation in the
implementation of the economic policies of Government of Nepal.

b) i) Promissory Note ii) Bills of Exchange


Section 2(f) of the Negotiable Instrument Act, 2034(1997) has defined the term as "Promissory
Note" means an instrument in writing except or Bank note containing an unconditional undertaking,
signed by the maker, to pay a certain sum of money to, or to the order of, a certain person or to the
bearer of the instrument. Provided that, this word shall not include the document prepared pursuant to
the Chapter of Transaction of the Muluki Ain (General Code). In such way, NRB Act, 2058 and
BAFIA, 2063 also defined this term in Section 2u and 2kk respectively.
This instrument should satisfy the elements of writing, express promise, unconditional, money
only, must be authorized with signature by the maker and certainty of the parties whether certain
person or bearer. In this document, parties are certain and should indicate in the face of the instrument.
Generally, two parties i.e. make- the person who makes the note and is known as the maker, and
payee- to whom the promise is made, are involved in the promissory note.
Therefore, promissory note, is, a written instrument prepared by the maker for the fulfillment of
own promise to pay certain sum of money to the certain person or bearer without condition.

ii)
Section 2(g) of the Negotiable Instruments Act, 2034(1997) has defined the term as "Bill of
Exchange" means an instrument in writing containing an unconditional order, signed by the maker,
directing a certain person to pay a certain sum of money to, or to the order of a certain person or to the

The Institute of Chartered Accountants of Nepal


15 of 62
Suggested Answers – Corporate and Other Laws
CAP II Examination – December 2012

bearer of the instrument in a certain date or after certain period of time or at the demand. In such way,
NRB Act, 2058 and BAFIA, 2063 also defined this term in Section 2t and 2jj respectively.
This instrument should also satisfy the certain elements i.e. writing, imperative order, certainty in
money, its amounts, date, period of time and parties, unconditional and must be authorized with
signature by the maker. Generally, there are three parties: like drawer or maker-who makes the BoE,
drawee-person to whom it is addressed and payee. Sometimes drawer and payee are same.
According to the above mentioned facts, Bills of Exchange are also a written instrument. It
contains a direction or order of a drawer to a certain person to pay a certain sum of money to the
person as specified in the instrument or to bearer.

c) Propriety Audit
The Auditor General of Nepal is considered five principles while conducting the audit of the government owned
corporations. One of these five principles is propriety audit.

Section 5 of the Audit Act, 2048 has defined matters to be audited in view of the propriety audit by the Office of the
Auditor General of Nepal.
(1) The Auditor General shall audit following matters considering the propriety thereof-

(a) On the propriety of any expenditure and its authorization, if in the opinion of the Auditor General such expenditure is a
reckless one or is an abuse of national property, whether movable or immovable, despite that the expenditure confirms to
the authorization, and

(b) On the propriety of all authorizations issued in respect of any grant of national property whether movable or
immovable, fixed or current, or underwriting of any revenue, or any contract, license or permits relating to mining, forest,
water resources, etc. and any other act of abandoning movable or immovable, assets of the nation.

(2) The Auditor General may not include in the report minor items of discrepancy and other items deemed as insignificant
in view of their property which were observed during the audit of income and expenditure.

The Institute of Chartered Accountants of Nepal


16 of 62
The Institute of Chartered Accountants of Nepal
Suggested Answers of Financial Management

CAP II Examination- December 2012

Working notes should form part of the answer. Make assumptions wherever necessary.

1. Following are the condensed Balance Sheets of Omega Ltd. for two years and the Statement of Profit and Loss
for one year:
Balance Sheet as at end of Ashadh
(Figures in Rs. „000)
2069 2068
Equity Share Capital 150 110
10% Redeemable Preference Shares 10 40
Capital Redemption Reserve 10 -
General Reserve 15 10
Profit and Loss Account balance 30 20
8% Debenture with convertible option 20 40
Other Term Loans 15 30
Total 250 250
Fixed Assets less Depreciation 130 100
Long Term Investments 40 50
Working Capital 80 100
Total 250 250
Statement of Profit and Loss
for the year ended 31st Ashadh, 2069
(Figures in Rs. „000)
Sales 600
Less: Cost of Sales (400)
200
Less:
Establishment Charges 30
Selling and Distribution Expenses 60
Loss on Sale of Equipment 15
Interest Expenses 5 (110)
90
Add:
Interest Income 4
Foreign Exchange Gain 10
Dividend Income 2
Damages received for loss of reputation 14 30
120
Less: Depreciation (50)
70
Taxes (30)
40
Dividends (15)
Net profit carried to Balance Sheet 25
Chief accountant of Omega Ltd. informed that ledgers relating to debtors, creditors and stock for both the years
were seized by the income tax authorities for the purpose of investigation and the same would not be available for
at least three months. However, he is able to furnish the following data:
(Figures in Rs. „000)
Ashadh end 2069 Ashadh end 2068
Dividend receivable 2 4
Interest receivable 3 2
Cash on hand and with bank 7 10
Investment maturing within two months 3 2
15 18
Interest payable 4 5
Taxes payable 6 3
10 8
Current ratio 1.5 1.4
Acid test ratio 1.1 0.8
It is also gathered that debenture-holders owning 50% of the debentures outstanding as on Ashadh end 2068
exercised the option for conversion into equity shares during the financial year ending on Ashadh 2069 and the
same was put through. Besides, an equipment was sold for Rs. 25,000 during the financial year 2068/69.
Suggested Answers – Financial Management
CAP II Examination – December 2012
Required:
Prepare a cash flow statement for the financial year 2068/69 under direct method. 20

Answer:
Cash Flow Statement
(Figures in Rs. ‘000)
A. Cash Flows from Operating Activities:
Cash Receipts from customers (WN 2) 621
Cash paid to suppliers and employees (WN 3) (406)
Operating expenses Paid (WN 4) (90)
Cash generated from operations 125
Income-tax paid (WN 5) (27)
Cash flow before extraordinary item 98
Add: Extraordinary items:
Foreign Exchange Gain 10
Damages for loss of reputation 14
Net cash from operating activities 122

B. Cash flows from Investing Activities:


Purchases of fixed assets (WN 6) (120)
Proceeds from sale of equipment 25
Proceeds from sale of investment (50 – 40) 10
Interest received (WN 7) 3
Dividend received (WN 7) 4
Net cash used in investing activities (78)

C. Cash flows from financing Activities:


Proceeds from issue of equity share capital (WN 8) 20
Redemption of Preference Share Capital (30)
Repayment of term loan (15)
Interest paid (WN 9) (6)
Dividend paid (15)
Net cash used in financing activities (46)
Net decrease in cash and cash equivalents (2)
Cash and cash equivalents at the beginning of the period (10 + 2) 12
Cash and cash equivalents at the end of the period 10

Working Notes: (figures in Rs.'000)


1. Determining the value of Current Assets and Current Liabilities 2068 2069
Current Ratio 1.40 1.50
Working Capital 100 80
Current Liabilities (100÷0.4) & (80÷0.5) 250 160
Current Assets (250+100) & (160+80) 350 240
Acid Test Ratio 0.8 1.10
Current Liabilities (as above) 250 160
Therefore, Quick Assets (QA) (CL×ATR) 200 176
Stock (CA – QA) 150 64
Other Current Assets (as given ) 18 15
(Dividend + Interest + Cash Equivalents)
Therefore, Debtors 182 161
(QA – Other Current Assets)
Therefore, Creditors 242 150
(CL – Interest payable – Taxes payable)
2. Cash Receipts from Customers
Sales (on accrual basis) 600
Add: Opening debtors (WN 1) 182 782
Less: Closing debtors (WN 1) - 161
Cash Received from Customers 621
3. Cash paid to suppliers
Cost of sales 400
Add: Opening creditors (WN 1) 242
Closing stock (WN 1) 64 706
Less: Closing creditors (WN 1) 150
Opening stock (WN 1) 150 - 300
406

The Institute of Chartered Accountants of Nepal


2 of 62
Suggested Answers – Financial Management
CAP II Examination – December 2012
4. Operating expenses Paid
Establishment Charges 30
Selling and Distribution expenses 60 90

5. Tax paid during the year


Tax payable in the beginning 3
Add: Provision for tax 30 33
Less: Tax payable at the end: -6
Tax paid during the year 27

6. Purchase of Fixed Assets


Balance at the end 130
Add: Depreciation for current year 50
Assets sold (Book Value ) (25 + 15) 40 220
Balance at the beginning 100
Purchase of Fixed Assets during the year: 120

7. Interest and Dividend received during the year


Interest Dividend
Opening Balance 2 4
Add: Accrued Income (Current Year) 4 2
6 6
Less: Closing Balance -3 -2
Received During the Year 3 4
8. Issue of Equity Share Capital for Cash
Capital at the end 150
Less: Capital issued to debenture-holders - 20 130
(Conversion- 50% of 40)
Less: Opening Capital - 110
Capital issued for Cash 20
9. Interest Paid during the year
Balance in the beginning 5
Add: Accrued during the year 5 10
Less: Balance at the end -4
Paid during the year 6

10. Cash and Cash Equivalents 2068 2069


Cash and Bank 10 7
Investments 2 3
12 10
11. Profit and Loss Account
Opening Balance 20
Profit for Current Year 25 45
Less: Transfer to General Reserve (15-10) 5
Transfer to Capital Redemption Reserve (10-0) 10 - 15
Closing Balance 30

12. The preference share capital in the beginning and at the end was Rs. 40 and Rs. 10 thousand respectively.
This redemption is backed by the issue of Equity Share Capital of Rs. 20 thousand and transfer of Profit and
Loss Account balance of Rs. 10 thousand to Capital Redemption Reserve Account.

2.
a) Albertine Ltd. has an investment opportunity available which will involve a capital outlay in each of the
next 2 years and which will produce benefits during the following 3 years. A summary of the financial implications
of this investment is given below:
Year Cash Flow (Rs. „000) Year Cash Flow (Rs. „000)
1 (1,000) 4 1,300
2 (1,000) 5 3,100
3 100
Albertine Ltd. currently has 100,000 shares in issue. The dividend just paid was Rs. 15 per share. In the absence of
the above investment, dividends are expected at this level for the next 3 years, but will then demonstrate perpetual
growth of 10 per cent per annum. The company is currently all equity financed and the required rate of return of the
equity investor is estimated to be 18 per cent.
The company has a long established policy of not using any debt finance and, because of the current depressed
state of the stock market, could not, in the near future, issue new equity. The only possible way of financing the
investment is, therefore, to reduce the dividend payments in the next 2 years. Cash received from the new
The Institute of Chartered Accountants of Nepal
3 of 62
Suggested Answers – Financial Management
CAP II Examination – December 2012
investment will all be distributed in the form of dividend. Growth in dividends at the rate of 10% will also be
maintained because of other operations.
Required: (3+5=8)
i) Calculate the current price of share of Albertine Ltd. when investment proposal is not accepted.
ii)Calculate the share price after the investment has been accepted using dividend valuation model, assuming that
the market knows of the dividend changes that will result from the investment.

b) As a part of the strategy to increase sales and profits, the sales manager of a fast moving consumer goods
(FMCG) company proposes to sell goods to a group of new customers with 10% risk of non-payment. This group
would require one and a half month credit and is likely to increase sales by Rs. 100,000 per annum. Production and
selling expenses amount to 80% of sales and the income-tax rate is 50%. The company‟s minimum required rate of
return (after tax) is 25%.
Required: (3.5+3.5=7)
i) Comment on the acceptance of the sales manager‟s proposal.
ii) Find the degree of risk of non-payment that the company should be willing to assume if the required rate of
return (after tax) were 30%.

Answer:
a)
i) Current price of Share (when investment proposal is not accepted)
The current market price of the share is the present value of expected future dividends discounted at the required
rate of return, i.e. 18%.Since the company is expected to pay a dividend of Rs. 15 for the next 3 years and
thereafter, the dividend will grow at the rate of 10%. The present market price with these parameters is ascertained
as below:
Dividend per year = Rs. 15
PVAF (at 18%, 3 years) = 2.174
Therefore, PV of dividends = Rs. 15 x 2.174 = Rs. 32.61
Price of share at the end of year 3 (P 3) = D4 = Rs. 15 (1+0.10) = Rs. 206.25
(with perpetual growth of 10%) ke – g 0.18 – 0.10
Present value of this amount at 18% for 3rd year = Rs. 206.25 x PVF(18%, 3)
= Rs. 206.25 X0.609 = Rs. 125.61
Present market price = Rs. 125.61 + Rs 32.61 = Rs. 158.22

ii) Current price of Share (when the investment proposal is accepted)


In first and second year the investment required of Rs.1,000,000 is financed by reducing the old dividend rate of
Rs.15 per share for 100,000 number of shares. And, thereafter all the cash flows from new investment is distributed
as additional dividend.
The present value of dividend under this situation will be as follows:
Year Old Dividend (Rs.) Change in Net Dividend PVIF @ PV
Dividend (Rs.) (Rs.) 18% (Rs.)
1 15 -10 5 0.847 4.24
2 15 -10 5 0.718 3.59
3 15 1 16 0.609 9.74
4 (15+10% of 15) =16.5 13 29.5 0.516 15.22
5 (16.5+10% of 16.5) = 18.15 31 49.15 0.437 21.48
Total 54.27
Price of share at the end of year 5 (P 5) = D6 = Rs. 18.15 (1 + 0.10) = Rs. 250 (with
perpetual growth of 10%) ke – g 0.18 – 0.10
Present value of this amount at 18% for 5th year = Rs. 250 XPVF(18%, 5)
= Rs. 250 X 0.437 = Rs. 109.25
Therefore, the market price under this situation = Rs. 109.25 + Rs. 54.27 = Rs. 163.52

(b)
i) Evaluation of Sales Manager's proposal Rs.
Additional sales from new customers per annum 100,000
Less: Risk of non-payment @ 10% 10,000
Net Turnover 90,000
Production and selling expenses (80% of sales) 80,000
Profit before tax 10,000
Income tax @ 50% 5,000
Profit after Tax 5,000
Average Investment in Debtors:
The credit period being 1 ½ months, there will be turnover of debtors of 8 times considering 12 month‟s year.
Thus, average debtors will be Rs. 100,000 / 8 = Rs. 12,500
Cost of goods sold being 80% of sales, the average investment in debtors would be 80% of Rs. 12,500, i.e. Rs.
10,000.
The Institute of Chartered Accountants of Nepal
4 of 62
Suggested Answers – Financial Management
CAP II Examination – December 2012
Thus, the rate of return (being PAT of Rs. 5,000) =
Rs. 5,000/ 10,000 x 100/10,000 = 50%
Since the company‟s minimum rate of return is 25%, the sales manager‟s proposal should be accepted.
Alternatively,
Cost of investment in debtor = 25% of Rs.10,000 = Rs.2,500
Increase in PAT = Rs. 5,000
Since, increase in PAT is greater than the additional cost of investment in debtor; the Sales Manager's proposal is
acceptable.

ii) Acceptable degree of risk of non-payment with the required rate of return (after tax) of 30%:
Particulars Rs.
Average investment in debtors 10,000
Required profit after tax @ 30% 3,000
Profit before tax (Grossed up by 50%) 6,000
Production and selling expenses 80,000
Required sales to achieved desired return 86,000
Additional sales 100,000
Therefore, acceptable degree of risk of non-payment 14,000
Acceptable degree of risk (in %) 14%

3.
a) South China Corporation is evaluating on investment projects for investment in new machinery to produce a
recently-developed product. The cost of the machinery, which is payable immediately, is Rs. 1.5 million, and the
scrap value of the machinery at the end of four years is expected to be Rs. 100,000. Capital allowances (tax-
allowable depreciation) can be claimed on this investment on a 25% reducing balance basis. Information on results
from the investment has been forecast to be as follows:
Year 1 2 3 4
Sales volume (units/year) 50,000 95,000 140,000 75,000
Selling price (Rs./unit) 25 24 23 23
Variable cost (Rs./unit) 10 11 12 12·50
Fixed costs (Rs./year) 105,000 115,000 125,000 125,000
This information must be adjusted to allow for selling price inflation of 4% per year and variable cost inflation of
2.5% per year. Fixed costs, which are wholly attributable to the project, have already been adjusted for inflation.
South China Corporation pays profit tax of 30% per year on one year in arrears.
South China Corporation has a nominal before-tax weighted average cost of capital of 12% and a nominal after-tax
weighted average cost of capital of 7%.
Required:
Calculate the net present value of the project and comment on whether this project is financially acceptable to
South China Corporation. 12

b) Consider the following information:


Stock A, Beta >1.
Stock B, Beta =1
Stock C, Beta <1.
Required: (2+1=3)
i) Explain the relation between the above stocks and their market.
ii) Explain, if you are a risk averter, in which stock would you invest.

Answer:
(a) Calculation of net present value (NPV)
As nominal after-tax cash flows are to be discounted, the nominal after-tax weighted average cost of capital of 7%
must be used.
Calculation of Net Present Value (Rs.)
Particular Year 1 2 3 4 5
Sales revenue WN 1 1,300,000 2,466,200 3,621,800 2,018,250
Variable costs WN 2 (512,500) (1,098,200) (1,808,800) (1,035,000)
Contribution 787,500 1,368,000 1,813,000 983,250
Fixed costs (105,000) (115,000) (125,000) (125,000)
Taxable cash flow 682,500 1,253,000 1,688,000 858,250
Tax liabilities (204,750) (375,900) (506,400) (257,475)
CA tax benefits WN 3 112,500 84,375 63,281 159,844
After-tax cash flow 682,500 1,160,750 1,396,475 415,131 (97,631)
Scrap value 100,000
Net cash flow 682,500 1,160,750 1,396,475 515,131 (97,631)
The Institute of Chartered Accountants of Nepal
5 of 62
Suggested Answers – Financial Management
CAP II Examination – December 2012
DF at 7% 0.9346 0.8734 0.8163 0.7629 0.7130
Present values 637,865 1,013,800 1,139,943 392,993 (69,611)

Present Value of Cash inflows = Rs.3,114,990


Cost of Machine = Rs.1,500,000
Net Present Value (NPV) = Rs.1,614,990

The project has a positive NPV of Rs. 1,614,990, so it is financially acceptable to South China Co. However, as this
is a recently-developed product, it may be appropriate to use a project-specific discount rate that reflects the risk of
the new product launch.

Working Note 1: Calculation of inflation adjusted Sales Revenue


Year 1 2 3 4
Price Inflation 100% 104% 108.16% 112.486% 116.985%
Selling price (Rs./unit) 25·00 24·00 23·00 23·00
Inflated selling price (Rs./unit) 26·00 25·96 25·87 26·91
Sales volume (units/year) 50,000 95,000 140,000 75,000
Sales revenue (Rs./year) 1,300,000 2,466,200 3,621,800 2,018,250

Working Note 2: Calculation of inflation adjusted Variable Cost


Year 1 2 3 4
Price Inflation 100% 102.5% 105.063% 107.69% 110.382%
Variable cost (Rs./unit) 10·00 11·00 12·00 12·50
Inflated variable cost (Rs./Unit) 10·25 11·56 12·92 13·80
Sales volume (units/year) 50,000 95,000 140,000 75,000
Variable costs (Rs./year) 512,500 1,098,200 1,808,800 1,035,000

Working Note 3: Calculation of capital allowance tax-benefits


Year 1 2 3 4
Capital Assets (Depreciation base) (Rs.) 1,500,000 1,125,000 843,750 532,812*
Depreciation Rate 0.25 0.25 0.25
Depreciation Amount (Rs.) 375,000 281,250 210,938 532,812
Tax Rate 0.30 0.30 0.30 0.30
Capital allowance tax benefits (Can be adjusted in 112,500 84,375 63,281 159,844
the year of relevant tax payable)
*Note: Capital base for 4th year = 843,750-210,938-100,000 = 532,812

(b)

i) The beta factor is a measure of a stock‟s volatility in terms of market risk. Therefore,

Stock A:
Where Beta> 1, the shares are described as aggressive; they outperform the market. This means they give a bigger
return than the market when the market return is positive and a bigger loss than the market, when the market return
is negative.

Stock B:
Where Beta= 1, the shares are described as neutral; their returns are in line with the average return of the stock
market.

Stock C:
Where Beta< 1, the shares are described as defensive; they are less risky than the market generally.

ii)A risk averter would generally invest in those stocks which are less risky than that of market. Therefore, Stock C
would be the choice for risk averters.

4.
a) PQ Limited currently has annual sales of Rs. 500,000 and an average collection period of 30 days. It is
considering a more liberal credit policy. If the credit period is extended, the company expects sales and bad debt
losses to increase in the following manner:
Credit Policy Increase in credit period Increase in sales (Rs.) Bad debts % of total Sales
A 10 days 25,000 1.2
B 15 days 35,000 1.5
C 30 days 40,000 1.8

The Institute of Chartered Accountants of Nepal


6 of 62
Suggested Answers – Financial Management
CAP II Examination – December 2012
D 42 days 50,000 2.2

The selling price per unit is Rs. 2. Average cost per unit at the current level of operation is Rs. 1.50 and variable
cost per unit is Rs. 1.20. The current bad debt loss is 1% of the total sales and the required rate of return on
investments is 20%. Ignore taxes and assume 360 days in a year.
Required:
Recommend the credit policy to be adopted. 8

b) KLS Limited has a total capitalization of Rs. 1,000,000 and it normally earns Rs. 100,000 before interest and
taxes. The finance manager of the company wants to take the decision regarding the capital structure. After a study
of the capital market, he gathers the following data:
Equity Capitalization Rate
Amount of Debt (Rs.) Interest rate (%) at given level of debt (%)
0 - 10.00
100,000 4.0 10.50
200,000 4.0 11.00
300,000 4.5 11.60
400,000 5.0 12.40
500,000 5.5 13.50
600,000 6.0 16.00
700,000 8.0 20.00

Assume that corporate taxes do not exist, and the firm always maintains its capital structure at book values.
Required: (4+3=7)
i) What amount of debt should be employed by the firm if the traditional approach is held valid?
ii) If the Modigliani-Miller approach is followed, what should be the equity capitalization rate?

Answer:
(a) The firm will maximize the shareholders value if it extends its period by additional 30 days (since expected
return is higher than required return). In fact, it can further relax credit period until its expected return becomes 20%
or net gain becomes zero.
Particulars Increase in credit period
Existing 10 days 15 days 30 days 42 days
A Credit period (days) 30 40 45 60 72
B Annual Sales (Rs.) 5,00,000 5,25,000 5,35,000 5,40,000 5,50,000
C Level of receivables 41,667 58,333 66,875 90,000 1,10,000
(at sales value) (AXB)/360 (Rs.)
D Incremental investment in receivables - 16,667 25,208 48,333 68,333
(C-41,667) (Rs.)
E Required incremental profit at 20% - 3,333 5,042 9,667 13,667
(0.20XD) (Rs.)
F Incremental contribution on additional sales - 10,000 14,000 16,000 20,000
@40% (2-1.2)/2 (Rs.)
G Bad debt losses (BX %bad debts) (Rs.) 5,000 6,300 8,025 9,720 12,100
H Incremental Bad debt losses - 1,300 3,025 4,720 7,100
(G-5,000) (Rs.)
I Incremental expected profit - 8,700 10,975 11,280 12,900
(F-H) (Rs.)
J Net Gain (I-E) (Rs.) - 5,367 5,933 1,613 (767)

Alternatively,
The investment in receivables can be calculated at cost. At current level of sales, the firm's average unit cost is Rs.
1.50. Since variable cost per unit is 1.20, we can find the fixed cost as follows:

Fixed Cost = Total Cost-Variable Cost


= (Rs.5,00,000) x 1.50/Rs. 2 - (Rs.5,00,000) x Rs.1.20/Rs.2
= Rs. 375,000 - Rs. 3,00,000
= Rs. 75,000

Thus, the total cost for different level of sales (assuming unit price and fixed cost do not change):
Sales (Rs.) Variable Cost (Rs.) Fixed Cost (Rs.) Total Cost (Rs.)
5,25,000 5,25,000 x 1.20/2 = 315,000 75,000 3,90,000
5,35,000 5,35,000 x 1.20/2 = 321,000 75,000 3,96,000
5,40,000 5,40,000 x 1.20/2 = 324,000 75,000 3,99,000
5,50,000 5,50,000 x 1.20/2 = 330,000 75,000 4,05,000
The Institute of Chartered Accountants of Nepal
7 of 62
Suggested Answers – Financial Management
CAP II Examination – December 2012

Investment in account receivables will be:


Investment in Receivables (Rs.) Changes in Investment (Rs.)
(3,75,000) x 30/360 = 31,250 -
(3,90,000) x 40/360 = 43,333 12,083
(3,96,000) x 45/360 = 49,500 18,250
(3,99,000) x 60/360 = 66,500 35,250
(4,05,000) x 72/360 = 81,000 49,750

The net gain from the credit policy can be re-calculated using incremental investment in accounts receivables at
cost. It would be higher now.
Particulars Increase in credit period
Existing 10 days 15 days 30 days 42 days
A Credit period (days) 30 40 45 60 72
B Incremental investment in receivables (Rs.) - 12,083 18,250 35,250 49,750
C Cost of investment at 20% (Rs.) - 2,417 3,650 7,050 9,950
D Incremental Bad debt losses (Rs.) - 1,300 3,025 4,720 7,100
E Incremental contribution on additional sales - 10,000 14,000 16,000 20,000
@40% (2-1.2)/2 (Rs.)
F Net Gain (E-D-C) (Rs.) - 6,283 7,325 4,230 2,950
In this case, the credit policy can be extended up to 42 days.

(b)
(i) As per the traditional approach, optimum capital structure exists when the weighted average cost of capital is
minimum. The weighted average cost of capital calculations at book value weights are as follows:

ke (1) We (2) kd (3) Wd (4) keWe (5) kdWd (6) k0 (7)=(5)+(6)


0.100 1.0 - - 0.1000 - 0.1000
0.105 0.9 0.040 0.1 0.0945 0.0040 0.0985
0.110 0.8 0.040 0.2 0.0880 0.0080 0.0960
0.116 0.7 0.045 0.3 0.0812 0.0135 0.0947
0.124 0.6 0.050 0.4 0.0744 0.0200 0.0944
0.135 0.5 0.055 0.5 0.0675 0.0275 0.0950
0.160 0.4 0.060 0.6 0.0640 0.0360 0.1000
0.200 0.3 0,080 0.7 0.0600 0.0560 0.1160

The company should employ debt of Rs.4,00,000 as the weighted average cost of capital is minimum at this level of
debt.

(ii) According to M-M approach, the cost of capital is a constant, and the cost of equity increases linearly with debt. The
equilibrium cost of capital is assumed to be equal to pure equity capitalization rate, which is 10% in the present
problem. The equity capitalization rate is given by the formula:
ke = k0+ (k0-kd) x Debt/ Equity

The equity capitalization rates would be:


Debt (Rs.) kd k0 (k0-kd) Debt/Equity ke ke Percentage
0 - 0.10 (0.10-0.000) - 0.1000 10.00
1,00,000 0.040 0.10 (0.10-0.040) 1,00,000/9,00,000 0.1067 10.67
2,00,000 0.040 0.10 (0.10-0.040) 2,00,000/8,00,000 0.1150 11.50
3,00,000 0.045 0.10 (0.10-0.045) 3,00,000/7,00,000 0.1236 12.36
4,00,000 0.050 0.10 (0.10-0.050) 4,00,000/6,00,000 0.1333 13.33
5,00,000 0.055 0.10 (0.10-0.055) 5,00,000/5,00,000 0.1450 14.50
6,00,000 0.060 0.10 (0.10-0.060) 6,00,000/4,00,000 0.1600 16.00
7,00,000 0.080 0.10 (0.10-0.080) 7,00,000/3,00,000 0.1467 14.67

5.
a) Mohan has just own a lottery and has three award options to choose from:
1. To receive a lump sum payment today of Rs. 61 million, or
2. To receive 10 annual end of year payment of Rs. 9.5 million, or
3. To receive 30 annual end of year payment of Rs. 5.5 million.
He expects to earn 8% annual return on his investment.
Required:
Recommend the best option for him. 5

The Institute of Chartered Accountants of Nepal


8 of 62
Suggested Answers – Financial Management
CAP II Examination – December 2012
b) PQR Limited deals in mines and geological survey. It is considering the following investment proposals for
mining:
Project Cash Flows
Year 0 Year 1 Year 2 Year 3
A -10,000 +10,000 Nil Nil
B -10,000 +7,500 +7,500 Nil
C -10,000 +2,000 +4,000 +12,000
D -10,000 +10,000 +3,000 +3,000
Assume Discount Rate of 10 per cent.
Required: (8+2=10)
i) Rank the projects according to each of the following methods:
Internal Rate of Return (IRR)
Net Present Value (NPV)
ii) Assuming the projects are independent, which one should be accepted? If the projects are mutually exclusive,
which project is the best?

Answer:
a) Calculation of present value of each option
1. Lump sum payment of Rs. 61 million; PV = Rs. 61 million
2. 10 annual end of year payment of Rs. 9.5 million;
PV= PMT x (PVIFA 8%, 10 years)
= Rs. 9.5 Million x 6.7101
= Rs. 63.75 Million
3. 30 annual end of year payment of Rs. 5.5 million;
PV= PMT x (PVIFA 8%, 30 years)
= Rs. 5.5 million x 11.2578
= Rs. 61.92 million
Since, option 2 provides highest present value, he should choose to receive 10 year annuity plan.

b) i) Cash Flows of Projects


Year A B C D
0 (10,000) (10,000) (10,000) (10,000)
1 10,000 7,500 2,000 10,000
2 - 7,500 4,000 3,000
3 - - 12,000 3,000

Discounted Cash Flows of Projects:


Year DF at 10% A B C D
0 1 (10,000) (10,000) (10,000) (10,000)
1 0.909 9,090 6,818 1,818 9,090
2 0.826 - 6,195 3,304 2,478
3 0.751 - - 9,012 2,253
NPV (910) 3,013 4,134 3,821
Rank IV III I II

IRR calculations:

Project A: The net cash proceeds in year 1 is just equal to the initial investment, therefore, IRR = 0%

For Project B, C & D Trying for NPV at DF of 40%


Year DF at 40% DCF of B DCF of C DCF of D
0 1 (10,000) (10,000) (10,000)
1 0.714 5,355 1,428 7,140
2 0.510 3,825 2,040 1,530
3 0.364 - 4,368 1,092
NPV (820) (2,164) (238)

IRR (B) = LR + × (HR – LR)

=10 + (40-10)

=10+23.58
= 33.58% (Rank =II)

The Institute of Chartered Accountants of Nepal


9 of 62
Suggested Answers – Financial Management
CAP II Examination – December 2012
IRR (C) =10 + (40-10)

=29.69% (Rank= III)

IRR (D) =10 + (40-10)

=35.83% (Rank=I)

ii) Between, the two time-adjusted (Discounted cash flow) investment criteria, NPV and IRR, NPV gives the
consistent results. If the projects are independent, either IRR or NPV method can be used since the same set of
projects will be accepted by any of the methods. In the present case, except Project A all the three projects should be
accepted if the discount rate is 10%. Under the assumption of 10% discount rate and mutually exclusive projects,
rankings according to IRR and NPV conflicts (except for Project A). If we follow the IRR rule, Project D should be
accepted. But the NPV rule says that Project C is the best.
Since the NPV rule gives consistent results in conformity with the wealth maximization principle, we would
therefore accept Project C following the NPV rule.

6. Write short notes on: (4×2.5=10)


a) Annuities and annuities due
b) Information asymmetry
c) Over the counter (OTC) market
d) The risk-return trade off

Answer:
a) Annuities and Annuities Due:
The term annuity refers to any terminating stream of fixed payments over a specified period of time. This usage is
most commonly seen in discussions of finance, usually in connection with the valuation of the stream of
payments, taking into account time value of money concepts, such as interest rate and future value.
Examples of annuities are regular deposits to a savings account, monthly home mortgage payments, and monthly
insurance payments. Annuities are classified by the frequency of payment dates. The payments (deposits) may be
made weekly, monthly, quarterly, yearly, or at any other interval of time.
An annuity-due is an annuity whose payments are made at the beginning of each period. Deposits in savings, rent
or lease payments, and insurance premiums are examples of annuities due.

b) Information Asymmetry
Information asymmetry deals with the study of decisions in transactions where one party has more or better
information than the other. This creates an imbalance of power in transactions which can sometimes cause the
transactions to go awry, a kind of market failure in the worst case. Examples of this problem are adverse selection,
moral hazard, and information monopoly. Most commonly, information asymmetries are studied in the context of
principal-agent problems. Information asymmetry causes misinforming and is essential in every communication
process.
Information asymmetry models assume that at least one party to a transaction has relevant information whereas the
other(s) do not. Some asymmetric information models can also be used in situations where at least one party can
enforce, or effectively retaliate for breaches of, certain parts of an agreement whereas the other(s) cannot.

c) Over the counter (OTC) market


Over-the-counter market functions as part of the secondary market for stocks and bonds not listed on a stock
exchange. The market is composed of brokers and dealers who stand ready to buy and sell securities at quoted
prices. Most corporate bonds and a growing number of stocks are traded over-the-counter instead of being traded on
an organized exchange.
The OTC market these days has become highly mechanized with market participants linked together by a
telecommunication network. Unlike the organized exchange, the participants of a OTC market do not come together
in a single place. A network is maintained and price quotations are made instantaneously. In the past, most
companies preferred to list their shares on major exchanges as a matter of prestige and necessity. This has undergone
change due to the rapid development in the field of electronics.

d) The Risk- Return Trade Off


This principle steps that potential return rises with an increase in risk. Low levels of uncertainty (low-risk) are
associated with low potential returns, whereas high levels of uncertainty (high-risk) are associated with high
potential returns. According to the risk-return tradeoff, invested money can render higher profits only if it is subject
to the possibility of being lost.
Because of the risk-return trade off, you must be aware of your personal risk tolerance when choosing
investments for your portfolio. Taking on some risk is the price of achieving returns; therefore, if you want to make
money, you can't cut out all risk. The goal instead is to find an appropriate balance - one that generates some profit,
but still allows you to sleep at night.
The Institute of Chartered Accountants of Nepal
10 of 62
Suggested Answers – Financial Management
CAP II Examination – December 2012

7. Distinguish between: (4×2.5=10)


a) Capital planning and Capital rationing
b) Treasury bills and Certificate of deposit
c) Permanent working capital and Variable working capital
d) Capital Assets Pricing Model and Arbitrage Pricing Model

Answer:
a) Capital Planning Vs. Capital Rationing
A proper plan for a company's capital expenditures is called Capital Planning. Capital expenditures are payments
made over a period of more than one year. They are used to acquire assets or improve the useful life of existing
assets; an example of a capital expenditure is the funding to construct a factory. Making a capital budget must
account for the potential profitability of the plans involved. Calculating the net present value or the internal rate of
return are two methods for determining a capital budget.
The act of placing restrictions on the amount of new investments or projects undertaken by a company is called
Capital Rationing. This is accomplished by imposing a higher cost of capital for investment consideration or by
setting a ceiling on the specific sections of the budget. Companies may want to implement capital rationing in
situations where past returns of investment were lower than expected.

b) Treasury bills and Certificate of deposits


Treasury bills are sold by the government on a discount basis. As a result, the investor does not get actual payment
of interest on the Treasury bills. The return to the investor is in the form of difference between the purchase price
and face (or par) value of the bill.
These bills are issued without the investor‟s name upon them, i.e. in the bearer form. This feature of the treasury
bills makes them easily transferable from one investor to another. The secondary market also exists for these bills
making them highly liquid. It is also considered risk-free since it has the backing and guarantee of the government.
Certificate of deposits (CDs) are marketable receipts for funds that have been deposited in a bank/financial
institution for a specified period of time. The funds thus deposited earn a fixed rate of interest. The denomination
and the maturities is agreed upon as per the needs and wishes of the investor.
Since the CDs are not sold at discount, the investor receives the amount deposited plus the interest earned thereon. A
secondary market also exists for the CDs. These may be issued in the registered or bearer form. The second types of
CDs are most common and popular due to their easy transferability and liquidity.

c) Permanent working capital Vs. Variable working capital


Permanent working capital is the minimum amount of gross working capital which is always maintained in spite of
the increase or decrease in the sales during the year. It comprises of the minimum cash balance, minimum inventory
level etc. If a firm does not face a seasonal cycle then it will have only a permanent working capital requirement.
Variable working capital is the amount of gross working capital that exceeds the amount of permanent working
capital at any time during the year. It is also important for the finance manager to decide sources for financing
seasonal current assets. The variable working capital is the result of the periodic fluctuations of the gross working
capital. For example, wheat mill may have higher inventories at the time of harvesting wheat. It causes the increase
in gross working capital.

d) Capital Assets Pricing Model Vs. Arbitrage Pricing Model


The capital asset pricing model (CAPM) states that the return on a stock depends on whether the stock's price
follows prices in the market as a whole. CAPM is useful because it is a statistical representation of past risk. Even
though past performance is no guarantee for future success there is a higher probability that a consistent past
performer will continue to do well over a new untested entry in the market.
Arbitrage pricing model (APM) holds that the expected return of a financial asset is largely based on its "beta".
Beta is the measure of the relationship between company related factors which influence financial performance and
the overall market in which the latter competes. Typically a company which has a beta of one will reflect the
market whereas a beta score of 0.75 means that a company will move up or down to the extent of 75 per cent of the
corresponding market movement.

The Institute of Chartered Accountants of Nepal


11 of 62
The Institute of Chartered Accountants of Nepal
Suggested Answers of Cost and Management Accounting

CAP II Examination- December 2012

1.
a) Rajat Chemicals Ltd., a company within the chemical industry, mixes powdered ingredients in two different
processes to produce one product. The output of Process 1 becomes the input of Process 2 and the output of Process 2 is
transferred to the packing department.
Following are the information pertaining to the week ended 11 th June 2012;
Process 1
Input:
Material A: 6,000 kilograms at Rs. 1 per kilogram
Material B: 4,000 kilograms at Rs. 2 per kilogram
Mixing Labour: 430 hours at Rs. 4 per hour
Normal Loss: 5% of weight input, disposed off at 32 paisa per kilogram
Output: 9,200 kilograms
There is no work-in-process at the beginning or end of the week.

Process 2
Input:
Material C: 6,600 kilograms at Rs. 2.50 per kilogram
Material D: 4,200 kilograms at Rs. 1.50 per kilogram
Flavoring Essence: Rs. 600
Mixing Labour: 370 hours at Rs. 4 per hour
Normal Waste: 5% of weight input with no disposal value
Output: 18,000 kilograms.
There is no work-in-process at the beginning of the week but 1,000 kilograms are in process at the end of the week and
estimated to be only 50% complete so far as labour and overhead were concerned.

Overhead of Rs. 6,400 incurred by the two processes is to be absorbed on the basis of mixing labour hours.

Required: (9+4+3=16)
i) Prepare Process 1 and 2 Account.
ii) Prepare Abnormal Loss Account and Packing Department Account.
iii) Calculate equivalent units produced.

b) Explain Economic Ordering Quantity. 4

Answer:
a)
i.
Dr. Process 1 Cr.
Per Kg. Per Kg.
Particulars Kg Rs. Rs. Particulars Kg Rs. Rs.

To Material A 6,000 1.00 6,000 By Normal Loss 500 0.32 160


By Abnormal
To Material B 4,000 2.00 8,000 Loss (W.N. 2) 300 2.00 600
To Mixing Labor (430
hours @ Rs.4) 1,720 By Transfer to
To Overhead(W.N. 1) 3,440 Process 2 9,200 2 18,400
Total 10,000 19,160 Total 10,000 19,160
Dr. Process 2 Cr.
Per Kg. Per Kg.
Particulars Kg Rs. Rs. Particulars Kg Rs. Rs.
To Process 1 9,200 2.00 18,400 By Normal Loss 1,000 - -
To Material C 6,600 2.50 16,500
To Material D 4,200 1.50 6,300 By Work-in-
To Flavoring Essence 600 progress (W.N.3) 1,000 - 2,320
To Mixing Labour (370
hours @ Rs.4) 1,480 By Packing
To Overhead (W.N.1) 2,960 Department 18,000 2.44 43,920
Total 20,000 46,240 Total 20,000 46,240
Suggested Answers – Cost and Management Accounting
CAP II Examination – December 2012
ii.
Abnormal Loss Account
Dr. Cr.
Per Kg. Per Kg.
Particulars Kg Rs. Rs. Particulars Kg Rs. Rs.
By Normal Loss 300 0.32 96

To Process 1 300 2.00 600 By Balance to P/L A/C 504


Total 300 600 Total 300 600

Packing Department Account


Dr. Cr.
Per Kg. Per Kg.
Particulars Kg Rs. Rs. Particulars Kg Rs. Rs.
To Process 2 18,000 2.44 43,920 By Balance C/D 18,000 2.44 43,920
Total 18,000 43,920 Total 18,000 43,920

iii. Statement of equivalent unit

Equivalent Units
Particulars Output Unit Material Labour Overhead
Completed 18,000 18,000 18,000 18,000
WIP (100% Material, 50%
Labour and Overhead) 1,000 1,000 500 500
Labour and Overhead - - - -
Normal Waste 1,000 - - -
Total 20,000 19,000 18,500 18,500

Working Notes:
1. Total overhead expenses : Rs. 6,400
Total labour hours in Process 1 and 2 = 800
Overhead absorption rate = Rs. 6,400/800 hours = Rs. 8 per labour hour
Overhead under Process 1 = 430 × Rs. 8 = Rs. 3,440
Overhead under Process 2 = 370 × Rs. 8 = Rs. 2,960

2. Cost of 9,500 Kg. of output is = ( Rs.19,160 – Rs. 160) i.e., Rs. 19,000
Hence cost per kg. of output is Re. 2.00

3. Cost per Equivalent Unit and WIP


Material = Rs. 41,800 / 19,000 = Rs. 2.20
Labour = Rs. 1,480 / 18,500 = Rs. 0.08
Overhead = Rs. 2,960 / 18,500 = Rs. 0.16

W.I.P.
Material = 1,000 × Rs. 2.20 = Rs. 2,200
Labour = 500 × 0.08 P = Rs. 40
Overhead = 500 × 0.16 P = Rs. 80
Rs. 2,320

b) Economic Order Quantity (EOQ): Purchase department in manufacturing concerns is usually faced with the problem
of deciding the „quantity of various items‟ which they should purchase. If purchases of material are made in bulk then
inventory carrying cost will be high. On the other hand if order size is small each time, then the ordering cost will be high.
In order to minimise ordering and carrying costs it is necessary to determine the order quantity which minimises these two
costs. The size of the order for which both ordering and carrying cost are minimum is known as economic order quantity.

Calculation of EOQ made as follows:


EOQ = √2AO/C
Where;
A= Annual requirement
O= Ordering cost per order
C= Carrying cost per unit per annum

At EOQ level total cost of material including ordering and holding cost will be minimal.

The Institute of Chartered Accountants of Nepal


13 of 62
Suggested Answers – Cost and Management Accounting
CAP II Examination – December 2012
2.
a) The Acme shelving Co. Ltd. manufactures shelving brackets in batches of 300. During May, Batch No. 23 was
machined at a rate of 15 per hour. Sixty of the brackets failed to pass inspection, but of these, 40 were thought to be
rectifiable. The remaining 20 were scrapped, and the scrap value was credited to the batch cost account. Rectification
work took nine hours. The following details are available for Batch No. 23:
Rs.
Raw materials per bracket 160
Scrap value per bracket 86
Machinists' hourly rate 420
Machine hour overhead rate (running time only) 360
Setting up of machine:
Normal machining 2,100
Rectification 1,800
Required: (4+6+2=12)
i) Calculate the cost of Batch No. 23 in total and per unit, if all units pass inspection.
ii) Calculate the actual cost of Batch No. 23 in total and per unit, after crediting the recovery value of the scrapped
components, and including the rectification costs.
iii) Calculate the loss incurred because of defective work.

b) In a manufacturing concern 20 workmen work in a group. The concern follows a group incentive bonus system
whereby each workman belonging to the group is paid a bonus on the excess output over the hourly production standard
of 250 pieces, in addition to his normal wages at hourly rate. The excess of production over the standard is expressed as a
percentage and two-thirds of this percentage is considered to be the share of the workman and is applied on the notional
hourly rate of Rs. 60 (considered only for the purpose of computation of bonus). The output data for a week are stated
below:

Days Man hours worked Output in pieces


Monday 160 48,000
Tuesday 172 53,000
Wednesday 164 40,000
Thursday 168 52,000
Friday 160 46,000
Saturday 160 42,000
Total 984 281,000

Required: (5+3=8)
i) Workout the amount of bonus for the week and the average rate at which each workman is to be paid.
ii) Compute the total wages including bonus payable to Ram who worked for 48 hours at an hourly rate of Rs. 25
and to Shyam who worked for 52 hours at an hourly rate of Rs. 30.

Answer:
a)
i. Calculation of the cost of Batch No. 23 in total and per unit, if all units pass inspection:
Batch No. 23
Particulars Rs.
Raw materials (300 X Rs.160) 48,000
Direct Labour:
Machinists' Cost (300/ 15 X Rs.420) 8,400
Setting up of machine:
Normal machining 2,100
Overhead (300/ 15 X Rs.360) 7,200
Total Cost 65,700
Per unit cost = Rs. 65,700/ 300 = Rs.219

ii. Calculation of the actual cost of Batch No. 23 in total and per unit, after crediting the recovery value of the scrapped
components, and including the rectification costs
Batch No. 23
Particulars Rs. Rs.
Raw materials (300 X Rs.160) 48,000
Less: Recovery value of scrap (20 X Rs.86) 1,720
46,280
Direct Labour:
Normal- Machinists' Cost (300/ 15 X Rs.420) 8,400
Rectification (9 hours X Rs.420) 3,780
12,180

The Institute of Chartered Accountants of Nepal


14 of 62
Suggested Answers – Cost and Management Accounting
CAP II Examination – December 2012
Setting up of machine:
Normal machining 2,100
Rectification 1,800
3,900
Overhead:
Normal (300/ 15 X Rs.360) 7,200
Rectification (9 hours X Rs.360) 3,240
10,440
Total Cost 72,800
Per unit cost = Rs. 72,800/ 280 = Rs.260

iii. Calculation of the loss incurred because of defective work


Batch No. 23
Particulars Rs.
Loss because of additional costs (Rs.72,800- Rs.65,700) 7,100
Loss because of faulty products (Rs.219 X 20 units) 4,380
Total loss incurred because of defective work 11,480

b)
(i)Computation of group incentive bonus
Man hours Standard output Actual output Excess % of excess 2/3% of excess Bonus (Rs.)
Days
worked (pcs) (W.N.1) (pcs) output (pcs) output (W.N.2) output for bonus (W.N. 3)
Monday 160 40,000 48,000 8,000 20.00 13.33 1,279.68
Tuesday 172 43,000 53,000 10,000 23.26 15.51 1,600.63
Wednesday 164 41,000 40,000 - - - -
Thursday 168 42,000 52,000 10,000 23.81 15.87 1,599.70
Friday 160 40,000 46,000 6,000 15.00 10.00 960.00
Saturday 160 40,000 42,000 2,000 5.00 3.33 319.68
984 5,759.69

Share of each individual workman: Rs. 5,759.69 / 984 = Rs. 5.85 per hour worked.

(ii)
Computation of wages to individual workmen
Ram Shyam
(a) Hours worked 48 52
(b) Hourly rate of payment Rs. 25 30
(c) Total wages at hourly rate (a) × (b) Rs. 1,200.00 1,560.00
(d) Incentive bonus (a) × Rs. 5.85 Rs. 280.80 304.20
(e) Total wages payable (c) + (d) Rs. 1,480.80 1,864.20

Working notes:
(1) Standard output for Monday (160 × 250) = 40,000 pieces and so on for other days.

(2) % of excess output for Monday (8,000 / 40,000 × 100) = 20% and so on for other days.

(3) Bonus for Monday (160 × 60 × 13.33%) = Rs. 1,279.68 and so on for other days.

3.
a) ABC Ltd. manufactures a single product and absorbs the production overheads at a pre-determined rate of Rs. 10 per
machine hour.
At the end of the financial year 2011-12, it has been found that actual production overheads incurred were Rs. 600,000. It
included Rs. 45,000 on account of written off obsolete stores and Rs. 30,000 being the wages paid for the strike period.
The production and sales data for the year 2011-12 is as under:
Production:
Finished goods 20,000 units
Work-in-progress (50% complete in all respect) 8,000 units
Sales:
Finished goods 18,000 units.
The actual machine hours worked during the period were 48,000 hours. It has been found that one-third of the under-
absorption of production overheads was due to lack of production planning and the rest was attributable to normal
increase in costs.

Required: (3+4+3=10)

The Institute of Chartered Accountants of Nepal


15 of 62
Suggested Answers – Cost and Management Accounting
CAP II Examination – December 2012
i) Calculate the amount of under-absorption of production overheads during the year 2011-12 and;
ii) Show the accounting treatment of under-absorption of production overheads
iii) Calculate the apportionment of under-absorption overheads over WIP, finished goods and cost of sales.

b) Deluxe Ltd. undertook a contract for Rs. 500,000 as on 1st July 2011. On 30th June 2012, when the accounts were
closed, the following details about the contract were gathered.
Rs.
Materials purchased 100,000
Wages paid 45,000
General expenses 10,000
Plant purchased 50,000
Materials on hand on 30th June 2012 25,000
Wages accrued on 30th June 2012 5,000
Work certified 200,000
Cash received 150,000
Work uncertified 15,000
Depreciation of plant 5,000

The above contract contained an escalation clause which read as follows.


In the event of materials and rates of wages increase by more than 5% the contract price would be increased accordingly
by 25% of the rise in the cost of materials and wages beyond 5% in each case.
It was found that since the date of signing the agreement, the prices of materials and wage rates increased by 25%. The
value of work certified does not take into account the effects of the above clause.
Prepare Contract Account. 10

Answer:
a) (i) Amount of under-absorption of production overheads during the year 2011-12:

Particulars Rs. Rs.


Total production overheads actually incurred 600,000
Less: Obsolete stores written off 45,000
Wages paid for strike period 30,000 75,000
Net production overheads actually incurred 525,000
Production overheads actually absorbed by 48,000 machine hours @ Rs. 10 480,000
Amount of under-absorption of production overheads 45,000

(ii) Accounting treatment of under absorption of production overheads:


It is given in the question that one third of the under absorbed overheads were due to lack of production planning and the
rest were attributable to normal increase in costs. So accounting treatment of under absorbed overheads will be as follows:
1. Under absorbed overheads due to lack of production planning being abnormal should be debited to the Profit &
Loss Account. Therefore, the amount to be debited to Profit & Loss Account is Rs. 45,000 × 1 / 3 = Rs. 15,000.
2. Under absorbed overheads attributable to normal increase in costs should be distributed over work-in-progress,
finished goods and cost of sales by using supplementary rate. The amount to be so distributed is Rs. 45,000 × 2 / 3 = Rs.
30,000.
(iii) Similarly as per question 20,000 units were completely finished and 8,000 units were 50% complete, apportionment
of unabsorbed overheads over work-in-progress, finished goods and cost of sales will be as follows:
Particulars Equivalent completed units Supplementary rate Rs.
Work-in-progress 4,000 1.25 5,000
Finished goods 2,000 (20,000-8000) 1.25 2,500
Cost of sales 18,000 1.25 22,500
24,000 30,000
Working notes:
Rs. 30,000
Supplementary rate per unit   Rs. 1.25
24,000
b)
Contract A/c for the Year Ended 30th June 2012
Dr. Cr.
Particulars Rs. Particulars Rs.
To Materials 1,00,000 By Work-in-progress A/c:
To Wages paid 45, 000 Work certified 2,00, 000
To Wages outstanding 5, 000 Work uncertified 15, 000
To General expenses 10, 000 Materials on hand 25, 000
To Depreciation of Plant 5, 000 Contract escalation * 5, 000
To Balance c/d – notional profit 80, 000
The Institute of Chartered Accountants of Nepal
16 of 62
Suggested Answers – Cost and Management Accounting
CAP II Examination – December 2012
Total 2,45, 000 Total 2,45, 000

To Profit and Loss A/c # 20, 000 By Balance b/d 80, 000
To Transfer to Reserve 60, 000
Total 80, 000 Total 80, 000

* Escalation:
Materials /wages increased by 25%
[a] Increase in material price [Rs.100000 – Rs.25000] 25/125 = Rs.15,000
[b] Increase in wages Rs.50,000 x 25/125 = Rs.10,000
Total Increase = [a] + [b] = Rs.25,000

This increase is 5% of the contract price, hence escalation clause apply. Escalation is 25% of the rise in the cost of
materials and wages beyond 5% in each case.

 Escalation = (25%-5%)25000=5,000

# Amount of profit to be credited to Profit and Loss A/c: As the contract is less than 50% complete, the following formula
will have to be used for computing the amount of profit to be credited to the Profit and Loss A/c:-

= 1/3 ×(Cash Received/Work Certified)  Notional Profit


= 1/3 × (Rs.1, 50, 000/2,00,000)  Rs.80,000
= Rs.20,000

4.
a) Metalica Trading Ltd. makes and sells a single product. The company‟s trading results for the year 2010 are as
follows :
Rs. ‟000
Sales 3,000
Direct materials 900
Direct labour 600
Overheads 900
Total cost 2,400
Profit 600

For the year 2011, the following are expected:


i) Reduction in the selling price by 10%
ii) Increasing in the quantity sold by 50%
iii) Inflation of direct material cost by 8%
iv) Price inflation in variable overhead by 6%
v) Reduction of fixed overhead expenses by 25%.

It is also known that;


i) In 2009, overhead expenditure totaled to Rs. 800,000.
ii) Total overhead cost inflation for 2010 has been 5% more than in 2009.
iii) Production and sales volumes have been 25% higher in 2010 than in 2009.

Required: (5+3+3=11)
i) Prepare a statement showing the estimated trading results for 2011.
ii) Calculate the break-even point for 2010 and 2011.
iii) Comment on the BEP and profits of the 2010 and 2011.

b) The following information is the extracted from the financial accounts of a manufacturing company for the last
financial year:
Rs.'000
Raw material consumed 5,000
Direct wages 3,000
Works overhead 1,600
Office overhead 700
Selling overhead 960
Bad debts 120
Legal charges 10
Interest received 120
Sales (120,000 units @ Rs.100) 12,000

The Institute of Chartered Accountants of Nepal


17 of 62
Suggested Answers – Cost and Management Accounting
CAP II Examination – December 2012
Closing inventory of WIP 240
Closing inventory of Finished goods (4,000 units) 320

The following information is extracted from the cost accounts for the same financial year:
Raw material consumed- Rs. 5,600,000
Recovery of works overhead- @ 20% on prime cost
Recovery of office overhead- @ Rs. 6 per unit of output
Recovery of selling overhead- @ Rs. 8 per unit sold.

Required: (6+3=9)
i) Prepare financial profit and loss account and Cost sheet for the financial year.
ii) Reconcile the difference in profit under the two sets of accounts.

Answer:
a) (i)
Statement showing trading results

Particulars 2010 2011


A. Sales: 3,000 4,050 (3,000 × 150% × 90%)
B. Less: Variable Costs: Direct material 900 1,458 (900 × 150% × 108%)
Direct labour 600 900 (600 × 150%)
Variable overhead (W.N.1) 300 477 (300 × 150% × 106%)
Total variable cost 1,800 2,835
C. Contribution [A – B] 1,200 1,215
D. Less: Fixed overheads 600 450 (600 × 0.75)
E. Profit [C – D] 600 765

(ii)
P/V Ratio = Contribution /Sales *100 = 1200/3000*100 1215/4050*100
= 40% = 30%
BEP = Fixed Cost / P/V Ratio = 600/40% = 450/30%
= Rs.1500 = Rs. 1500
(iii)
Particulars 2010 2011 % change
BEP 1500 1500 No Change
Fixed overheads 600 450 150/650*100 = 25%
P/V Ratio 40% 30% 10%/40%*100= 25%
Profit 600 765 -165/600*100=- 27.5%

Both fixed cost and P/V ratio have declined by 25% equally. So, BEP sales remain the same. The contribution is only Rs.
1,215 in 2011 though quantity is increased by 50%. This is due to increase in production cost and decrease in selling
price. This is more than made up by decrease in fixed cost so that overall profit has increased by 27.5%.

Working notes:
1. Calculation of variable overheads and fixed overheads
Total overheads for same production in 2010 = 800 × 105% = 840
Variable overheads for 2010 = (900-840)/ (125-100) *125 = 300
Fixed overheads for 2010 = 900 – 300 = 600

b) Solution:
i.
Financial Profit & Loss Account
Particulars Rs.'000 Particulars Rs.'000
To Raw material consumed 5,000 By Sales 12,000
To Direct wages 3,000 By Closing inventory:
To Works overhead 1,600 WIP 240
To Office overhead 700 Finished goods 320
To Selling overhead 960 By Interest received 120
To Bad debts 120
To Legal charges 10
To Net profit 1,290
12,680 12,680

Cost Sheet
For the financial year
The Institute of Chartered Accountants of Nepal
18 of 62
Suggested Answers – Cost and Management Accounting
CAP II Examination – December 2012
Particulars Rs.'000 Rs.'000
Raw material consumed 5,600
Direct wages 3,000
Prime Cost 8,600
Works overhead (20% on Prime cost) 1,720
10320
Less: Closing WIP (240)
Works Cost 10,080
Office overhead (Rs.6 *124,000) 744
Cost of production 10,824
Less: Closing Finished goods (10,824*4000/124000) (349)
Cost of goods sold 10,475
Selling overhead (Rs.8 * 120,000) 960
Cost of sales 11,435
Net profit 565
Sales 12,000
Note: Unit produced = Sold unit + closing inventory

ii.
Reconciliation Statement
Particulars Rs.'000 Rs.'000
Profit as per financial accounts 1,290
Add:
Closing inventory of finished goods overvalued in cost accounts 29
Bad debts not charged in cost accounts 120
Legal charges not charged in cost accounts 10 159
1449
Less:
Raw material overcharged in cost accounts 600
Works overhead over absorbed in cost accounts 120
Office overhead over absorbed in cost accounts 44
Interest received not included in cost accounts 120 884
Profit as per cost accounts 565

5. Distinguish between: (4×2.5 =10)


a) Efficiency Audit and Propriety Audit
b) Cost Control and Cost Reduction
c) Controllable Cost and Uncontrollable Cost
d) Running Charges and Maintenance Charges

Answer:
a) Cost audit, apart from having all the normal ingredients of audit, i.e. vouching, verification etc., has within its
domain elements of efficiency audit and propriety audit. Efficiency audit is directed towards the measurement of whether
corporate plans have been effectively executed. It is concerned with the utilisation of the resources in economical and
most remunerative manner to achieve the objectives of the concerns. It comprises of studying the plans of the
organization, comparing actual performance with plans and investigating the reasons for variances to take remedial action.
For example, the effective utilization of capital in an organization can be gauged by determining the return on capital
employed.
On the other hand propriety audit is concerned with the executive actions and plans bearing on the finances and
expenditure of the company. The cost auditor has to judge:
1. Whether the planned expenditure is designed to give optimum results;
2. Whether the size and channels of expenditure were designed to produce the best results; and
3. Whether the return from expenditure on capital as well as current operations could be bettered by some other
alternative plan of action.

b) Cost control is operated through setting standards of targets and comparing actual performance therewith with a
view to identifying deviations from standards or norms and taking corrective action in order to ensure that future
performance conforms to standards or norms. It is a preventive action.

Cost reduction, on the other hand, is a continuous process of critical cost examination, analysis and challenge of
standards. It is a corrective action. Each aspect of the business viz. products, process, procedures, methods, organization,
personnel, etc is critically examined and reviewed with a view to improving the efficiency and effectiveness and reducing
costs. Even in an organization where efficient cost control is in operation, there is always room for cost reduction. In cost
reduction there is a permanent reduction in per unit cost where such permanency is lacking in cost control.
The Institute of Chartered Accountants of Nepal
19 of 62
Suggested Answers – Cost and Management Accounting
CAP II Examination – December 2012

Cost control lacks the dynamic approach which planned cost reduction demands. In a cost reduction plan, standards which
are the basis of cost control are constantly challenged for improvement.

Budgetary control and standard costing are essential tools and techniques of cost control. There are several distinct tools
and techniques of cost reduction such as value engineering, work study, standardization, simplification, variety reduction,
quality measurement and research, operations research, market research, job evaluation, merit awards, incentives,
improvement in design, mechanization, automation etc.

c) Controllable cost and uncontrollable cost


Controllable cost Uncontrollable cost

i. Controllable cost are the costs which can be influenced i. Uncontrollable cost are the costs which can not be
by action influenced by action
ii.
ii. Controllable cost are influenced by specified member ofiii. Uncontrollable cost are not influenced by specified
an undertaking member of an undertaking
iii. iv.
iv. Can be influenced by the action of executive of each v. Can not be influenced by the action of executive of each
responsibility centers responsibility but could avoid with initial decision
vi.
v. vii. Can not be minimized for cost control purpose
Can be minimized for cost control purpose viii.
vi. ix. Generally Fixed cost are in the nature of uncontrollable
vii. Generally variable cost and semi variable cost are in the cost
nature of controllable cost

d) Running charges and maintenance charges both are the parts of total costs in operating costing. These are used to
prepare a cost statement in an industry where services are rendered and goods are not produced.

Running charges are variable costs relating to rendering of services, where as maintenance charges are generally in the
nature of semi-variable or semi-fixed costs. The items to be included in running charges and maintenance charges depend
on the nature of the operation. For example, cost of fuel in transport sector, cost of food items in restaurant are running
charges; besides the cost of repairs and spares in transport sector and the cost of lighting, consumable stores etc. in hotels
are maintenance charges. To find out per unit cost of services rendered, the costs are required to be segregated into
variable and fixed costs. Therefore, the maintenance charges require to be segregated into variable and fixed costs

6. Answer the following questions: (4×2.5=10)

a) Briefly explain the circumstances that warrant the need for preparation of Flexible budget.
b) Discuss ABC analysis as a technique of inventory control.
c) Explain the role of uniform costing in inter-firm comparison.
d) What do you mean by cost driver? Explain with example.

Answer:
a) A 'Flexible Budget' is defined as 'a budget which, by recognizing the difference between fixed, semi-variable and
variable costs is designed to change in relation to the level of activity attained'. In a fixed budgetary control, budgets are
prepared for one level of activity whereas in a flexible budgetary control system, a series of budgets are prepared for a
number of alternative production levels or volumes. Flexible budgets represent the amount of expenses that is reasonably
necessary to achieve each level of output specified. In other words, the allowances given under flexible budgetary control
system serve as standards of what costs should be at each level of output.

The need for the preparation of the flexible budgets arises in the following circumstances:
(i) Seasonal fluctuations in sales and/or production, for example in soft drink industry;
(ii) A company which keeps on introducing new products or makes changes in the design of its products frequently;
(iii) Industries engaged in make-to- order business like ship building;
(iv) An industry which is influenced by changes in fashion; and
(v) General changes in sales

b) It is a system of inventory control. It exercises discriminating control over different items of stores classified on
the basis of investment involved. Usually they are divided into three categories according to their importance, namely,
their value and frequency of replenishment during a period.

„A‟ category of items consists of only a small percentage i.e. about 10% of total items handles by the stores but require
heavy investment about 70% of inventory value, because of their high price or heavy requirement or both.
The Institute of Chartered Accountants of Nepal
20 of 62
Suggested Answers – Cost and Management Accounting
CAP II Examination – December 2012
„B‟ category of items are relatively less important – 20% of the total items of material handled by stores and % of
investment required is about 20% of total investment in inventories.
„C‟ category – 70% of total items handled and 10% of value.

For „A‟ category items, stocks levels and EOQ are used and effective monitoring is done.
For „B‟ category same tools as in „A‟ category are applied.
For „C‟ category of items, there is no need of exercising constant control. Orders for items in this group may be placed
after 6 months or once in a year, after ascertaining consumption requirement.

c) When several undertakings in an industry, group and association use same costing principles and practices, it is
known as uniform costing. The application of uniform costing in an industry provides the means whereby relevant
information can be obtained and it helps in comparing the production efficiency of two firms at the time of merger.

Inter-firm comparison is the technique of evaluating the performance efficiencies, costs and profits of firms in an industry.
The application of uniform costing greatly facilitates inter-firm comparison by providing information relating to costs,
profits, prices, efficiency etc. Inter-firm comparison will not be possible if uniform costing is not in existence.

d) A cost driver is a variable, such as the level of activity or volume, which causally affects costs over a given time
span. That is, there is a cause-and-effect relationship between a change in the level of activity or volume and a change in
the level of total costs. Cost drivers signify factors, forces or events that determine the costs of activities. Costs drivers are
the links and they can link a pool of costs in an activity center to the product. For example, if product design costs change
with the number of parts in a product, the number of parts is a cost driver of product design costs. Similarly, miles driven
are often a cost driver of distribution costs.

The cost driver of a variable cost is the level of activity or volume whose change causes proportionate changes in the
variable cost. For example, the number of vehicles assembled is the cost driver of the total cost of steering wheels. If setup
workers are paid an hourly wage, the number of setup hours is the cost driver of total (variable) setup costs.

Costs that are fixed in the short run have no cost driver in the short run but may have a cost driver in the long run.
Consider the cost of testing quality of paints in a paint manufacturing company. These costs consists of testing department
equipment and staff costs that are difficult to change and, hence, are fixed in the short run with respect to changes in the
volume of production. In this case, volume of production is not a cost driver of testing costs in the short run. In the long
run, however, the paint manufacturing company will increase or decrease the testing department's equipment and staff to
the levels needed to support future production volumes. In the long run, volume of production is a cost driver of quality
testing costs.

The Institute of Chartered Accountants of Nepal


21 of 62
The Institute of Chartered Accountants of Nepal
Suggested Answers of Business Communication and Marketing

CAP II Examination- December 2012

Section -'A'

1. Read the following case carefully and answer the questions given below: 10
Imagine that you are the Secretary of the CA Association and the latter is planning to hire three Researchers to conduct some
research studies. You have been given the responsibility to conduct the final interview of the ten short-listed candidates.
While interviewing one of the candidates, the door was not closed properly and your mobile phone rang with a loud ring
tone. While making an attempt to turn it off you forgot the question you were about to ask. Also, as the door was open, you
could not hear properly what the candidate was speaking, as the talks among other candidates waiting in the lobby was too
loud. After the interview, the President of the organization called you for a meeting to discuss about the ten short-listed
candidates you interviewed to select the final three.

a) As an interviewer, you should have either switched off your mobile phone or kept it in silent mode. If you were the
candidate giving the interview, what things should you have done while being interviewed for employment?
b) Noise such as the mobile ring and talks among other candidates are some of the common barriers to communication.
What other barriers could affect the interpersonal communication between you and the candidate?
c) What do you understand by information and analytical report? In your meeting with the President, what information
will include in the report?

Answer:
a) Don‟t talk too much during the interview.
-Listen carefully to what the interviewer is asking and do not interrupt the interviewer in the middle of the conversation.
-Dress appropriately and be careful with the body language (sitting postures, use of hands during communication).
-Be confident – gather necessary information about the organization before interview

b) Barriers to the communication between interviewer and interviewee:


-The use of jargon, over-complicated or unfamiliar terms.
-Differences in perception and viewpoint.
-Language differences and the difficulty in understanding unfamiliar accents.
-Expectations and prejudices which may lead to false assumptions or stereotyping. People often want to hear what they
expect to hear rather than what is actually said and jump to incorrect conclusions.
-Cultural differences. lying between and among the interlocutors.
c)
-Informational reports: The reports, which contain functional information with regard to a specific business activity, data,
facts, feedback, and other types of information, are called informational reports.
-Analytical reports: The reports which contain interpretations, analysis and comments of the report writer are called analytical
report. These offer both information and analysis, and they can also include recommendations.
-Information to be included:
-Background of the interview, number of applicants, details of all short-listed candidates for final interview, their strengths
and weaknesses, the points each of them have scored, suggestions on who should be selected and why.

2. What are the most significant barriers to interpersonal communication? Use examples to elaborate your points
wherever possible. 10

Answer:
You can improve your chances of communicating successfully by learning to recognize barriers that are known to disrupt the
process. The most significant barriers for individuals are bypassing, differing frames of reference, lack of language skill, and
distractions.
Bypassing. One of the major barriers to disrupt communication involves words. Each of us attaches a little bundle of
meanings to every word, and these meanings are not always similar. Bypassing happens when people miss each other with
their meanings. Let‟s say your boss asks you to “help” with a large customer mailing. When you arrive to do your share, you
learn that you are expected to do the whole mailing yourself. You and your boss attached different meanings to the word help
. Bypassing can lead to major miscommunication because people assume that meanings are contained in words. Actually,
meanings are in people. For communication to be successful, the receiver and sender must attach the same symbolic
meanings to their words. A recent study revealed a high likelihood of miscommunication when people use common but
vague words such as probably, always, never, usually, often, soon, and right away. What do the words really mean?
Differing Frames of Reference. Another barrier to disrupt communication is your frame of reference . Everything you see
and feel in the world is translated through your individual frame of reference. Your unique frame is formed by a combination
of your experiences, education, culture, expectations, personality, and other elements. As a result, you bring your own biases
Suggested Answers – Business Communication and Marketing
CAP II Examination – December 2012
and expectations to any communication situation. Because your frame of reference is different from everyone else‟s, you will
never see things exactly as others do. American managers eager to reach an agreement with a Chinese parts supplier, for
example, were disappointed with the slow negotiations process. The Chinese managers, on the other hand, were pleased that
so much time had been taken to build personal relationships with the American managers. Wise business communicators
strive to prevent miscommunication by being alert to both their own frames of reference and those of others.
Lack of Language Skill. No matter how extraordinary the idea, it won‟t be understood or fully appreciated unless the
communicators involved have good language skills. Each individual needs an adequate vocabulary, a command of basic
punctuation and grammar, and skill in written and oral expression. Moreover, poor listening skills can prevent us from
hearing oral messages clearly and thus responding properly.
Distractions. Other barriers include emotional interference, physical distractions, and digital interruptions. Shaping an
intelligent message is difficult when one is feeling joy, fear, resentment, hostility, sadness, or some other strong emotion. To
reduce the influence of emotions on communication, both senders and receivers should focus on the content of the message
and try to remain objective. Physical distractions such as faulty acoustics, noisy surroundings, or a poor cell phone
connection can disrupt oral communication. Similarly, sloppy appearance, poor printing, careless formatting, and
typographical or spelling errors can disrupt written messages. What‟s more, technology doesn‟t seem to be helping.
Knowledge workers are increasingly distracted by multitasking, digital and information overload, conflicting demands, and
being constantly available digitally. Clear communication requires focusing on what is important and shutting out
interruptions.

3. Conflicts within an organization need to be settled as early as possible. Suggest five specific ways of conflict
resolution within a business organization. 10
Answer:
Conflicts are very obvious features of organizational management. These are the result of various aspects such as
misunderstanding, egos, etc. If the conflicts are not managed properly they can harm the organization very badly. However, if
the conflicts are carefully managed they are more advantageous rather than harmful. They have positive results too. So,
instead of lingering them they need to be settled as early as possible. For this, very careful and appropriate strategies that fit
the given context are required to be adopted. Some of the specific ways of conflict resolution are:

 Proaction: Addressing minor conflicts before they become serious.


 Communication: Bringing the conflicting parties to a negotiating table to discuss the issues.
 Research: Identify the factual reasons of the conflict.
 Flexibility: Create a flexible environment for free and open discussion.
 Fair Play: Encourage everyone to work out a fair solution to the problem.

4. In the international scene, knowing and identifying cultural differences can make a big difference between
successful business negotiations and humiliating rejections. Elaborate. What are some of the effective international business
communication guidelines? 10

Answer:
International business communication plays a major role in this, as knowing how to communicate through their point of view
is a great advantage. Each country has their own way of saying things, what may not be offending to you may be offending to
them. International business communication is all about knowing the important thing that lies behind peoples‟ words in the
international arena. International business communication is communicating across cultures and the first thing you must
always put in mind is the basic understanding that one size doesn‟t fit all. Always putting in mind that the cultural practices
or habits you have does not mean that everyone else around the globe does.
The most important and worrying variables in the international scene of business are the cultural differences that most
international business executives fail to comprehend resulting in most international business slip-ups. The failure to identify
and adapt to these differences through effective international business communication can mean the difference between a
closed deal and a faceless failure. Assuming cultures affect the way people communicate with each other and language above
all else, is the core of culture.
Culture is the different ways in which people behave, act, or think towards themselves, families, and other people in the
society. It includes customs connected with social, political, family, and religious values. Different cultures have deviating
values for time and its use, for personal space, and for family values and chats. These differences must be identified and
recognized to bridge the gaps between cultures and for real communication to take place.
Learning effective international business communication is communicating across cultures, and it takes great sensitivity and
awareness by studying cultures and becoming more perceptive and adaptable in communication efforts.
Some of the effective international business communication guidelines that may help you are:
Listening with an open mind
Communicating across cultures is all about putting aside personal feelings and listening deeply to the person talking. Develop
your listening skills and learn how to hold your tongue. Knowing your own culture and the others culture as well will give

The Institute of Chartered Accountants of Nepal


23 of 62
Suggested Answers – Business Communication and Marketing
CAP II Examination – December 2012
you a better chance of bridging the differences. Keeping an open mind and sensitivity to these differences is a key factor for
better communication.
Non-verbal and Verbal behavior
Every language had their own linguistic preferences, and these preferences will give clues about those particular cultures
behavior, manners, and thinking. The English use modesty and understatement, and are sometimes purposely unclear to avoid
confrontation and very polite. Eloquence, expressiveness, and flowery of words are most common among Italians and
Spaniards, Germans on the other hand are very logical in manner. Knowing what people mean when they say yes is very
important; it may mean yes, I understand but not yes, I agree.
Gestures
Gestures involves sensitivity and awareness, as gestures that are not offending to you might be very offending to the other, as
in the case of beckoning someone to come over, some would hold out their hand with the palm up and then move the fingers
gesturing the person to come. In Asian culture however, this gesture is considered rude. To gesture someone to come in
Asian culture should be done with the palm facing

5. Briefly explain any FOUR of the following: (4×2.5=10)


a) Conducting an effective presentation
b) A resume
c) Advantages of conflict in team
d) Analyzing data for reports
e) Merits and demerits of grapevine form of communication

Answer:
a) Conducting an Effective Presentation.
Before conducting a presentation, you should always prepare a backup plan. Make an extra copy of your work for backup and
print your speaking notes. Use colour highlighters to mark the key words on the notes. This will help you keep your place as
you speak.
If you have a complex presentation with different multimedia, practise ahead of time. Produce extra copies of the handout
materials. Check out the room ahead of time, and make sure it has everything you need. When presenting your work, use the
following guidelines:
i) Be sure to introduce yourself and any co-presenters to your audience.
ii) Speak slowly and clearly!
iii) Familiarise the audience with a background explanation of your study.
iv) Proceed in a logical manner, telling what you did step by step.
v) Point to graphs, charts, etc., when you refer to them.
vi) Give credit to those whom you have contacted and to those who have helped you.
vii) When you finish speaking, ask the audience if there are any questions they would like to ask.
viii) Make sure that the questions have been addressed and thank the audience for any suggestions.
ix) Thank the audience for attending.

b) Resume
Resume is also known as bio-data. It is the formal arrangement of an individual‟s personal details. It is prepared in a given
pattern. Generally, it includes personal details, academic qualification, work experience, training, awards, etc. The resume
should be as impressive and persuasive as possible so that the employer could be convinced to offer a job to the presenter.
The resume is found in various formats. However, the purpose and the contents are almost similar. It includes systematically
personal details and its main purpose is to convince the employer for job.

c) Advantages of conflict in team


Conflict is a normal part of every workplace and every team. Although the word alone is enough to make our heart begin to
thump, conflict is not always negative. When managed properly, conflict can improve decision making, clarify values,
increase group cohesiveness, stimulate creativity, decrease tensions, and undermine dissatisfaction. Unresolved conflict,
however, can destroy productivity and seriously reduce morale.

d) Analyzing data for reports


Analysis and interpretation of the data is an important phase of preparing report and thesis from a study, or a survey, or a
research. When the data are collected from various sources by using various primary and secondary tools, these data are to be
processed and analyzed. Through the analysis and interpretation of data we can prepare the report with convincing findings
and recommendations. The data analysis procedures also vary significantly according to the nature of the study and report.
Generally, the data can be analyzed by using different statistical tools, scales and figures. We can use percentage, mean,
median, standard deviation and other statistical tools and test scores for the analysis of data. The tables, diagrams and figures
are obviously used. From such analysis comparison, correlation can be explicitly shown.

e) Merits and demerits of the grapevine form of communication:


The Institute of Chartered Accountants of Nepal
24 of 62
Suggested Answers – Business Communication and Marketing
CAP II Examination – December 2012
Merits
- Speedy transmission: It transmits information very speedily.
- Feedback value: The managers or top bosses of an organisation get the feedback regarding their policies, decisions
memos etc. Feedback reaches much faster.
- Support to other channels: It is a supplementary or parallel channel of communication
- Psychological satisfaction: It gives immense psychological satisfaction to the workers and strengthens their solidartiy,
demerits
- It is less credible. It cannot always be taken seriously.
- It does not always carry the complete information.
- If often distorts the picture or often misinforms.

Section -'B'
6. Read the following case carefully and answer the questions given below:
Wheel, a product of Uniliver, is a detergent that caters to the needs of the mass market. It was first launched by Hindustan
Lever in 1987 in India. Wheel Green is the single largest selling detergent brand in Nepal by value. Wheel is packed with
powerful Lather that cleans even the toughest stains on collars and cuffs with case. It cleans effectively with lesser effort,
making washing light and easy. Moreover, Wheel does contain a high percentage of soda.
Wheel also has a premium variant called „Active Wheel‟. It gives consumers the value of three benefits in one. It not only
cleans effectively with less effort but also keeps the colored clothes looking bright and has a fresh fragrance. Active Wheel
gives consumers “quality clean and care” at an affordable price. Involvement of lesser number of middlemen, limited
distribution outlets and weak promotion base has adversely affected its desired level of sales and profit.
In short, Wheel can be summed up as –“Mehenat Kam, dhulai No. 1”. Wheel is, therefore, the smart housewife‟s choice.
Questions:
a) Make a short analysis of the case and describe product attributes of Wheel. 5
b) Give suggestions to Uniliver Ltd. for improving the performance of Active Wheel. 5

Answer
a) Wheel, a product of unilever is found very popular in Nepal. Many people liked wheel for their laundry needs as it
has many good qualities. It is treated as less harmful to hands and comparatively of greater quality and economical. It has
good prospect in Nepalese market. Clear with less efforts, product compositions and good results are its notable features due
to which it is preferred by many people.
Major attributes of wheel detergent are: cleans efficiently with lesser effort, does not burn hands or harm clothes, many
benefits in one-makes clothes bright, fresh fragrance, affordable price, etc.
b) The above attributes have made wheel a popular detergent units less effort and good results. In other words
(mehanat kam dhulai No.1) wheel has several good features leading to be highly – demanded product. Washes clothes clean
and customer care at a reasonable price. Unilever has to increase number of middlemen, improve its promotional base by
using new advertising, publicity, sales promotion strategy so that it can expand more market to improve the performance of
Active Wheel.

7.
a) Define marketing. Explain the importance of marketing to the society. 5
b) Explain the fundamental principles of societal marketing. 5

Answer
a) A few definition of marketing are as under:
Stantion, Etzen and walker have difined:
“Marketing is a total sysyem of business activities designed to plan, price, promote and distribute want –satisfying products
to target markets to active organizational objectives”
Philip Katter:
“Marketing is a societal process by which individuals and groups obtain what they need and want through creating, offering
and freely exchanging products and services of value and others.”
From the above definitions we can define marketing as a societal process and all activities to create exchange relationships
for customer need fulfillment and achieve organizational goals in the given environment. So all the activities directed in the
flow of good and services from producer to consumer, come under marketing.

b) The societal marketing concept assumes that along with the consumers' wants and interests, social interests too
should be considered so that long-run social welfare could be maintained. The underlying premises of the societal marketing
concept are:
i. society's long-run interests;
ii. Consumers will increasingly favor an organization which shows a concern with meeting their wants, long-run
interests, and society's long run interests;
The ultimate goal of marketing should be to achieve organizational through social goods welfare and well being.
The Institute of Chartered Accountants of Nepal
25 of 62
Suggested Answers – Business Communication and Marketing
CAP II Examination – December 2012

8.
a) What makes the market segmentation meaningful and effective? Explain. 5
b) What do you understand by Marketing Information System (MIS)? Explain marketing research as a major part of MIS.
5

Answer
Market segmentation is the process of dividing large, heterogeneous (dissimilar) markets into smaller, homogeneous (similar)
sub-markets. In other words, market segmentation is the process by which an organization attempts to match a total
marketing program to the unique needs of one or more customer groups in the marketplace.
The basic objective of segmenting a market involves improving competency, sales volume, and profitability of the
organization. To achieve this objective market segmentation must be effective and meaningful. In order to make market
segmentation effective and meaningful the following conditions or must be fulfilled or followed.
 There should be similarities in customer characteristics
in Each Segment
 Customer Differences should be Measurable or Identifiable
 Segments should be Accessible or Reachable
 Segments should be large Enough
 Segments should be sufficiently Stable
a) Marketing information system (MIS) is an on-going process consisting of people and equipments, to generate, sort,
analyze, evaluate and distribute needed accurate information in time for use in marketing decision marketing.
Organizations need to design and use an effective MIS to gather information and manage its continued flow to support in
achieving marketing objectives. It consists of input →processing →output →feedback components. Use of information
technology especially the computer and internet play an important role in MIS.
Marketing research system is an important component of MIS. Marketing research is a systematic and objective study of
problems pertaining to the marketing of goods and services. Marketing research is done to provide information to
management so that it can identify to react to marketing opportunities and problems. It also provides the required, reliable
and timely information for making marketing decisions to managers. Most large companies have their own marketing
research department. So marketing research specifics the information required, collecting information, analyzing the results
and communicating the findings. As such marketing research is very valuable to MIS.

9.
a) Point out the new product development process and explain test marketing. 5
b) What is physical distribution? Explain the contributions of physical distribution system in marketing. (2+3=5)

Answer
a) Stages in new product development are as fellows:
- Idea generation
- Idea screening
- Concept dev. & testing
- Marketing strategy and business analysis
- Product development
- Test marketing and
- Commercialization.
Test marketing:
Product is manufactured in small volume for test marketing. With a view to see consumer response on product it is placed on
sale in a test market. It is send with brand name, packaging and all other procedures followed at a limited scale.
Test market helps to correct/ improve the marketing mix. Is a product's test market is ok, its goes to final stage of
commercialization. At this stage trial, first repeat, adoption and purchase frequency are monitored.
b) Physical distribution is concerned with the management of physical flow of goods from the points of suppliers to
the points of purchasers.
The main objective of physical distribution is getting the right products safely to the right places at the right time at the least
possible cost.
The main contributions of physical distribution system are as follows:
a. It provides customer service by managing transportation, warehousing whenever and wherever required;
b. It distributes goods more safely at right time by selecting the proper means of transportation and physical handling.
c. It minimizes the total cost by selecting the right channel, transportation system and proper materials handling
technique; and
d. It supplies goods to the right target market by delivering the desired goods to target customers.
e. It fulfills customers‟ needs and provides them satisfaction because the actual movement and delivery of goods is
possible only through the physical distribution system.

The Institute of Chartered Accountants of Nepal


26 of 62
Suggested Answers – Business Communication and Marketing
CAP II Examination – December 2012
10. Briefly explain the following: (5×2=10)
a) E-marketing
b) Marketing environment in Nepal
c) Stages of consumer buying process
d) Mark-up pricing method
e) Branding

Answer
a) E-marketing
E-marketing describes a wide variety of electronic services used in marketing. Purchase of goods and selling are both done
through electronic media under E-marketing. Direct markets use media to make direct offers to potential buyers. Besides
other electronic data interchange through the use of fax, e-mail, use of ATM and the use of internet and online services are
gaining popularity. The internet today functions as an information source, an entertainment source, a communication
channel, a transaction and distribution channel. Any one having internet service cam send e-mail, exchange views, shop for
produces, access news, and exchange business information. So internet provides marketing opportunities which come under
E-marketing.

b) Nepal is a small landlocked Himalayan country situated at the heart of Asia. It is situated between India in the west,
south and east and the Tibet Autonomous Region of China in the north. Nepal has a multi-party system.
The major environmental forces that have impact on Nepalese market and trade pattern are economic environment,
demographic environment, political-legal environment, technological environment, natural environment, and competitive
environment.
Nepal is an agrarian country. Contribution of agriculture sector to the country‟s real GDP and employment is high. Farming
is still highly labour-intensive.
Recently, marketing opportunities for services sectors like computer software, insurance, banking, education, hospitals and
nursing homes have been growing rapidly.
The demographic character of Nepal shows that the market structure of the country is diverse in nature.
Nepal is a labour-abundant country. Labour is comparatively cheap in Nepal. Industrial development has not taken proper
direction in Nepal, so the use of appropriate technology is still shy. Rural farming is still characterized by traditional local
technology.
New innovations are lacking in Nepal. Almost all the technologies transferred into Nepal are “me too” type of technology
i.e., “imitating” one. Recently, information technology has been gaining momentum in the country in commercial sector.
For the last several years, Nepal has been badly affected by political unrest in the country, which has multiplier effects on the
country‟s economic and social development.

c) Consumer behavior is concerned with the study of consumers‟ activities and actions relating to the selection,
purchase and consumption of goods.
Though there is no hard and fast rule regarding the compulsion to follow up a rigid buying decision process, a rational
consumer usually follows.
buying process of five steps:
 Need recognition
 Information search
 Evaluation of alternatives
 Purchase decision
 Post purchase evaluation

d) Mark-up pricing
Price is the value paid on what is exchanged and measured in monetary units. It is the act of determining the exchange value
between the purchasing power and utility or satisfaction derived though the purchase of products.
Companies are having different objectives, policies and methods of setting prices their products. Out of many commonly
used methods of pricing is cost oriented pricing. Mark-up pricing method is to add a standard mark-up to the product cost.
It per unit cost is Rs.80 and 25 percent mark up then manufacture will charge Rs 100 for the product.

e) Branding
Branding is the art and cornerstone of marketing. A brand is a name, term, sign, symbol or design or a combination of them,
intended to identify the goods or services of one seller or group of sellers and to differentiate them from those of competitors.
So, a brand identifies the producer or seller. A brand can be better positioned by associating its name with a desirable benefit.
Companies promote their brand for different purposes depending upon what they want to prefer quality, performance, safety,
adventure, emotion, etc. So, a brand is essentially a marketer‟s promise to deliver a specific set of features, benefits and
services to the buyers. Brands are legally protected and superior to trade marks. A good brand name requires all such
qualities. Brand names differ with the company. Today, branding is such a strong force that hardly anything goes unbranded.

The Institute of Chartered Accountants of Nepal


27 of 62
The Institute of Chartered Accountants of Nepal
Suggested Answers of Income Tax and VAT

CAP II Examination- December 2012

1. Answer the following with reference to the Indian Income Tax Act, 1961:

a) Is there any income not chargeable to tax in case of Electoral Trust? Are there any conditions to be satisfied? Explain. 2

b) Mr. Phil comes to India for the first time on September 1, 2011. On September 15, 2011, he joins a company on monthly
salary of Rs. 90,000, as a part-time consultant (duty hours: 6.30 pm to 9.30 pm). Prior to September 15, 2011, Mr. Phil does
not have any source of income. On October 9, 2011, he starts a trading business in computer hardware after obtaining
approval of his employer. For the previous year March 31, 2012, he has the following income:
Salary from the part-time employment: Rs. 585,000; income from the business of trading in computer hardware in India: Rs.
1 million; and foreign income from the same business: $ 50,000. Find out the residential status of Mr. Phil for the assessment
year 2012-13. 6

c) Mr. Jury, a foreign citizen, is residing in India since 2004. While completing his assessment for the assessment year 2010-
11, on February 14, 2011, the Assessing Officer comes to know that Mr. Jury will leave India on April 12, 2012 with no
intention of returning. Is there any provision wherein Assessing Officer can make assessment in assessment year 2010-11? 2

Answer:
a) Voluntary contribution received by an electoral trust is treated as income under section 2(24)(iia). By virtue of section 13B,
donation received by an electoral trust will not be chargeable to tax if the following conditions are satisfied;

i. The electoral trust is approved by the Central Board of Direct Taxes in accordance with the scheme made by the Central
Government.
ii. The electoral trust will have to distribute to political parties 95 percent of the aggregate donations received by it during the
previous year along with the surplus, if any, brought forward from any earlier years. For this purpose, a political party means
a political party registered under section 29A of the Representation of the People Act, 1951.

iii. The electoral trust functions in accordance with the rules made in this regard by the Central Government.

b) As per section 6(1), an individual is said to be resident in India in any previous year, if he satisfies at least one of the
following conditions (basic conditions)-
i. He is in India in the previous year for a period of 182 days or more.
ii. He is in India for a period of 60 days or more during the previous year and 365 days or more during 4 years immediately
preceeding previous year.
Further, as per section 6 (6), a resident individual is treated as “resident and ordinarily resident” in India if he satisfies the
following two conditions (additional conditions)-
i. He has been resident in India in at least 2 out of 10 previous years immediately preceeding the relevant previous year
ii. He has been in India for a period of 730 days or more during 7 years immediately preceeding the relevant previous year.

For the assessment year 2012-13, Mr. Phil has the following sources of income in India:
Sources of Income Previous Year Number of days when Mr. Phil
was in India
Salary Income September 15, 2011 to March 31, 2012 199 days
Business Income October 9, 2012 to March 31, 2012 175 days

For the first source on income, Mr. Phil becomes resident in India by satisfying one of the basic conditions. As he comes to
India for the first time in 2011, he is unable to satisfy any of the additional conditions. Thus, he is a resident but not ordinary
resident in India for the first previous year.
For the second source of income, Mr. Phil is a non-resident, as he satisfies none of the basic conditions.
It may be noted that he is non-resident in India for the business income and resident but not ordinarily resident for the salary
income. In view of section 6(5), if a person is resident in India for one of the sources of income, he will be deemed to be
resident in India for all other sources of income in the same assessment year. In respect of the assessment year 2012-13, Mr.
Phil will, therefore, be regarded as resident but not ordinarily resident for all sources of income.
Suggested Answers – Income Tax and VAT
CAP II Examination – December 2012
c) Section 174 prescribes that if it appears to the Assessing Officer that an individual may leave India during the current
assessment year or shortly thereafter with no intention of returning to India, the total income of such individual up to the
probable date of his departure from India shall be chargeable to tax in that assessment year.

Mr. Jury has intention of leaving India only on April 12, 2012 which is not shortly after then the current assessment year as
prescribed by section 174. Hence, in assessment year 2010-11, the assessing officer can make assessment of previous year
2009-10. The assessing officer can assess the income of assessment year 2011-12/2012-13 (For the period April 01, 2012-
April 12, 2012) in assessment year 2011-12 as per section 174.

2.
a) Anubhav & Co. has taken on rent the premises from Buddhi Shree & Co for the operation of its business which pays
monthly rent of Rs. 100,000. Anubhav & Co. is deducting withholding tax at the rate of 10% amounting to Rs. 10,000 per
month and making payment amounting to Rs. 90,000 per month to Buddhi Shree & Co. While submitting Income Tax
Return, tax liability of Buddhi Shree & Co, to be payable during Income Year 2068/69 came to Rs. 200,000. Answer the
following question mentioning the relevant provision of Income Tax Act, 2058.
i) What is the amount of tax to be deposited?
ii) Will your answer be different if tax liability of Buddhi Shree & Co comes to Rs. 100,000 instead of Rs. 200,000?
iii) What is the implication if Buddhi Shree & Co had not adjusted the amount of withholding tax paid through Anubhav &
Co. while submitting income tax return? 8

b) Mr. „Z‟ a Canadian citizen is deputed by the Canadian government to work on a Canadian aided project and he stays in
Nepal for 94 days. He receives his salary in Canada and he is provided a free accommodation and a daily allowance of Rs.
2,000 to meet his boarding and other expenses.

i) Examine the liability to tax under Income Tax Act, 2058.


ii) Will your answer be different if Mr. „Z‟ comes to Nepal under the services of a private contractor instead of the Canadian
government in the above case? 4

c) The status of property, plant and equipments and repair and improvement Expenditure XYZ & Co. during Income Year
2068/69 is as follows;
Block of Assets Repair and Improvement Expenditure (Rs.) Depreciation basis (Rs.)
Building Block 2,000,000 100,000,000
Computer Block 100,000 800,000
Automobile Block 1,600,000 16,000,000
Total 3,700,000 116,800,000
Answer the followings mentioning the relevant provisions of Income Tax Act, 2058;

i) Can repair and improvement expenditure of Rs. 37, 00,000 be claimed under Income Tax Act, 2058. If not, compute the
amount of repair and improvement expenditure that can be claimed by XYZ & Co. during Income Year 2068/69.
ii) What is the implication of the amount of repair and improvement expenditure, if any, which cannot be claimed during
Income Year 2068/69? 8

Answer:
a) Section 93 (2) and (3) of Income Tax Act, 2058 prescribes the followings;
1) In case tax has been deducted from payments other than those not subject to final tax deduction, the person whose tax has
been deducted shall be deemed to have paid tax as follows:
(a) The tax amount deducted from payments under Section 87, 88 or 89.
b) The tax amount under Sub-Section (3) of Section 90, or the tax amount which is to be deemed to have been deducted
from payments, deposited, if any, at the Department by the advance tax deducting person or the person whose tax is
deducted.

2) The person whose tax is deducted under Sub-Section (2) may make a claim for its adjustment only with the amount of tax
payable by him in the income year in which the payment has been made.
Accordingly,
i. The amount of tax liability to be deposited by Buddhi Shree & Co, while submitting income tax return is Rs.80,000 after
adjusting the withholding tax deducted and paid through Anubhav & Co (Rs. 10,000*12 as TDS on rent).
ii. In this case, Buddhi Shree & Co can make a claim for the carry forward in next year or claim for the refund under section
113 of the Act, for the amount which is in excess of amount paid as withholding tax (i.e. Rs. 20,000).

The Institute of Chartered Accountants of Nepal


29 of 62
Suggested Answers – Income Tax and VAT
CAP II Examination – December 2012
iii. If Buddhi Shree & Co had not adjusted the amount of withholding tax paid through Anubhav & Co. during Income Year
2068/69 it cannot claim for the adjustment in forthcoming year. It can claim for the excess tax paid as refund under section
113 of the Act.

b)
i. The income derived by a natural person from the employment of public service of foreign country is exempted under
section 10 of Income Tax Act, 2058. Following conditions shall have to be satisfied in order to get the exemption;
a. Such natural person is resident or non-resident of Nepal because of employment only; and
b. Such payments shall be made from the public fund of foreign county.
Hence Mr. „Z‟ satisfies the above conditions, his salary and allowances are exempt from income tax in Nepal.
The place of receiving the salary of allowances is immaterial for the tax purpose.

ii. If Mr. „Z‟ comes to Nepal under the services of a private contractor instead of the Canadian government, the salary
received in Canada and the allowances received in Nepal combined together shall be taxable in Nepal.
The private contractor (employer) has to deduct withholding taxes from his salary as per section 87. In case the contractor
does not withhold the tax at source and does not deposit the same to revenue department, the contractor and Mr. „Z‟ are
responsible to pay the tax jointly or severally. As his total stay in Nepal is of 94 days only, he is non-resident in Nepal for the
Income Year and he has to pay tax at the rate of 25% on the taxable salary without allowing for initial deductions.

c) Section 16 of Income Tax Act, 2058 prescribes the following provisions;


1) Expenses incurred during and Income Year on repair and improvement of an asset owned and used depreciable asset in
order to generate income, is allowed to be deducted from taxable income.

2) Expenses on repair and improvement regarding a block of owned and used assets during an Income Year in excess of 7%
of the depreciable basis of the respective block at the end of Income Year, cannot be deducted during the Income Year.
3) The portion of the expenses disallowed during the year is allowed to be capitalized to the carrying amount of the
respective block of the assets,
Accordingly,
i. The entire amount of repair and improvement of Rs. 37,00,000 cannot be claimed. The amount of repair and improvement
that can be claimed under section 16 of Income Tax Act, 2058 can be computed as below;
Block of Assets Dep basis (Rs.) 7% of Dep basis (Rs.) Actual Repair and Improv Exp (Rs.) Claimable Expenses (Rs.)
Building Block 100,000,000 7,000,000 2,000,000 2,000,000
Computer Block 800,000 56,000 100,000 56,000
Automobile Block 16,000,000 1,120,000 1,600,000 1,120,000
Total 116,800,000 8,176,000 3,700,000 3,176,000

ii. The following is the implication of the amount which is in excess of the limit prescribed under section 16(2).
Block of Assets Claimable Expenses (Rs.) Amount in excess of limit (Rs.) Implications of the amount in excess of the limit
Building Block 20,00,000 - As the entire expense is claimable, no implication
Computer Block 56,000 44,000 The excess amount shall be included in the
opening depreciation basis of the following IY.
Automobile Block 11,20,00,0 4,80,000 The excess amount shall be included in the
opening depreciation basis of the following IY.
Total 31,76,00,000 524,000

3.
a) Mr. Ram had purchased a Land on 2063.10.01 for Rs.1 Crore. On 2066.05.06, he divorced his wife Mrs. Seeta. As part of
his divorce settlement, he transferred the land to his ex- wife Seeta on 2066.05.10. At the time of transfer, the market value of
the land was Rs. 1 Crore 40 Lacs. During the transfer, Mrs. Seeta incurred Rs. 10,000 towards registration and other legal
charges. Mr. Ram had informed the details of the transfer of land to ex-wife and elected the application of section 43 in
writing to Inland Revenue Office. Mrs. Seeta has sold the land on 2066.10.05 for Rs. 1 Crore 50 Lacs. By mentioning the
relevant provision of section 43, calculate the gain or loss on disposal of the land in the hands of both Mr. Ram and Mrs.
Seeta. If they do not elect the application of this section, how the incomings and outgoings are calculated? 7

b) Mr. Z , retired person from Nepal Government on Ist Sharwan, 2068 after 30 years of Service. After retirement, he joined
Kathmandu Bank at Kathmandu on the same day, .i.e. Sharwan, 2068 and remained in service till Ashad end 2069.
Information about his income are as follows:
S.N. Particulars Nepal government Kathmandu bank
1. Salary Rs. 3,00,000 Per month
2. House rent allowance Rs. 20,000 Per month
3. Other allowance Rs. 10,000 Per month
The Institute of Chartered Accountants of Nepal
30 of 62
Suggested Answers – Income Tax and VAT
CAP II Examination – December 2012
4. PF contribution Additional 10 % of basic salary & grade
5. Pension received Rs. 144,000 during the year ----------------

Other information:
 He has been provided a vehicle for personal as well as official use by the Kathmandu bank.
 Both employers have deducted 10 % of salary for provident fund and deposited 20 % to the approved retirement fund.
 He had received all retirement payment in Ashad 2068.
 He has not been provided Dashai allowances during the period.
 His wife does not have any income in this Income year and they declared they are couple.

Calculate the assessable income, taxable income and tax amount of Mr.Z for the income year. 6

c) Mr. Ram had purchased a Land situated at Bhaktapur on 2062.05.01 for Rs. 50 lacs.During the extension of Arniko
Highway, Nepal Government took possession of the land with a notice of compulsory acquisition on 2066.05.07. As
compensation, Nepal Government has given a sum of Rs. 80 Lacs to Mr. Ram. Mr. Ram purchased another land
approximately 2 Kms. away on 2067.04.05 for Rs. 60 Lacs. Mr. Ram has requested in writing to Inland Revenue Office for
the application of section 46 of the act. Compute the gain or loss on disposal of land in the hands of Mr. Ram by mentioning
the relevant provisions of the Act. Will the amount of gain or loss differ if he has not elected for application of this section? 7

Answer:
a) As per section 43 of the Act, in case a property is transferred due to a divorce settlement or to one's husband, wife, ex-
husband or ex-wife and election has been done for the application of this section, then:
- The net outgoings incurred by the transferor will be treated as if they have been received from the transfer and
- The same amount will be treated as net outgoings for the transferee.
Since election has been done for the application of section 43, gain or loss is calculated as follows:

In the hands of Mr. Ram


Incomings Rs. 1 Crore.
Less: Outgoings Rs. 1 Crore
Gain or Loss Nil
The transferor experiences neither a gain nor loss from the disposal of the property.

In the hands of Mrs. Seeta


Incomings Rs.1,50,00,000
Less:
Outgoings:
Cost of the transferred asset1,00,00,000
Registration and legal charge 10,000 Rs.1,00,10,000
Gain from disposal of land Rs. 49,90,000
If they do not elect the application of this section, then the market value of the property or the actual outgoings, whichever is
higher are treated as incomings for the transferor and outgoings for the transferee.

Thus, if they do not elect sec 43 to be applicable in this case the gain shall be calculated as under:
In the hands of Mr. Ram
Incomings Rs. 14,000,000
Less: Outgoings Rs. 10,000,000
Gain or Loss Rs. 4,000,000
The transferor experiences Rs. 40 lakhs as gain from the disposal of the property.
In the hands of Mrs. Seeta
Incomings Rs.15,000,000
Less:
Outgoings:
Deemed Cost of the transferred asset14,000,000
Registration and legal charge 10,000 Rs.14,010,000
Gain from disposal of land Rs. 990,000

b) Calculation of Assessable Income, taxable income and tax

Particulars Rs.
Salary 3,600,000

The Institute of Chartered Accountants of Nepal


31 of 62
Suggested Answers – Income Tax and VAT
CAP II Examination – December 2012
House rent Allowance 240,000
Other allowance 120,000
Addition on Provident fund 360,000
Vehicle facilities (0.5 % of Salary) 18,000
Pension 144,000
Assessable Income 4,482,000
Less: Contribution to Provident fund 300,000
Whichever is lower
Actual contribution Rs. 720,000
One third of Assessable income Rs. 1,494,000
Upper limit as per section 63 Rs. 300,000

Taxable Income 4,182,000

Less Exemption on pension (25% of basic exemption limit) 50,000

Taxable Income after exemption 4,132,000

Tax calculation

Slab Tax Rate Tax Rs.


200,000 1% 2,000
100,000 15% 15,000
38,32,000 25% 9,580,000
Additional tax (see note) 168,200
Total annual tax for Mr. Z 1,143,200

Note:
Taxable income 4,182,000
Less: 2,500,000
Taxable income over
than Rs. 2,500,000 1,682,000
40% of tax @25% 168,200

c) Section 46 of the Act envisages the circumstances when a person has to dispose of an asset or liability due to orders issued
by government or due to changes in legal decisions or when a person has to dispose of an asset or a liability due to forceful
conditions. The conditions applicable for availing this section are as under:
a. There must be a case of involuntary disposal of an asset or a liability.
b. The asset or liability is exchanged for another same kind of asset or liability within one year of the disposal.
c. The person requests in writing for application of this section for the disposal with replacement.

Section 46 further states that the total incomings for the transferred assets shall be the sum of the following amounts:
- The net outgoings for the assets up to the disposal of the asset and
- In case the amount received from involuntary acquisition exceeds the amount paid or payable for the replaced asset, the
excess of the amount.
In the given case,
1. Compensation received from Government Rs.8,000,000
2. Cost of the Replaced asset Rs. 6,000,000
3. Excess of 1-2 Rs. 2,000,000
4. Net Outgoings for the asset before disposal Rs. 5,000,000
5. Total Incomings (3+4) Rs. 7,000,000
6. Total Outgoings Rs. 5,000,000
7. Gain from Disposal Rs. 2,000,000

If he has not selected for application of this section, then the gain is calculated as under:
Incomings Rs. 8,000,000
Less:
Outgoings Rs. 5,000,000
Gain from disposal Rs. 3,000,000

The Institute of Chartered Accountants of Nepal


32 of 62
Suggested Answers – Income Tax and VAT
CAP II Examination – December 2012
4. ABC Ltd. has entered into a construction contract with PQR Ltd. with a term of 3 years. The contract price and the total
estimated cost are Rs.60 Billion and Rs. 40 Billion respectively in Year1 of the contract, cost amounting to Rs. 10 Billion
was incurred and up to year 2 Rs.25 Billion has been expensed. Calculate the income of ABC Ltd. to be included in the
concerned head of income with respect to long term contract in Year 1 and Year 2 by also showing the calculation of
Cumulative Inclusions and Cumulative Deductions.
Briefly explain on Long Term Contract, Excluded Contract and Deferred Return. (5+5=10)

Answer:
Year 1
Percentage of completion of the contract = 10 Billion/40 Billion =25%.
Cumulative inclusion is derived by multiplying total estimative cumulative inclusions up to the completion by percentage of
contract completed.
Rs. 60 Billion * 25% = Rs. 15 Billion.
Cumulative Deductions is derived by multiplying total estimated deductions up to completion by percentage of contract
completed.
Rs. 40 Billion * 25% = Rs. 10 Billion.
Amount to be included as income in Year 1 = Rs. 15 Billion – Rs. 10 Billion
= Rs. 5 Billion.
Year 2
Percentage of completion of contract = 25/ 40 = 62.5%.
Cumulative inclusions = Rs.60 Billion * 62.5% = Rs. 37.5 Billion
Cumulative Deductions = Rs. 40 Billion * 62.5% = Rs. 25 Billion
Amount to be included in year 2 as income = Rs. 37.5 – Rs.25-Rs.5 = Rs. 7.5 Billion.

Long Term Contract – As per section 26(1) of the act, a long term contract is a contract for production, installation,
construction or the services related to them, which runs for more than twelve months. To establish a long term contract under
this section, there should be a deferred return as a condition of the contract and the contract should not be an excluded
contract.
Excluded Contract – As per Rule 11 of the Income Tax Rules, Excluded contract is any contract created by reason of an
interest in an entity or by obtaining a membership in a retirement fund or any contract of investment insurance. Excluded
contract is not taken as a long term contract.
Deferred Return - As per Rule 10 of the Income Tax Rules, a contract shall be called as deferred return contract if any party
to a contract does not declare the information related to the estimated profit and estimated loss for the period of every six
months starting from the commencement of the contract as required by IRD.

5.
a) EFG Ltd. has given a contract to Garibdas construction, a party not registered in VAT, for construction of a Shopping
Complex built exclusively for commercial purposes. The cost of the shopping complex is Rs.55 lacs. By mentioning the
relevant provision of the act and rules, comment on the taxability of the transaction. 4

b) Moonlight Traders is engaged in the business of selling Television sets on installment basis. As per the policy and
contract agreement, forty percent of the Price of the Television Rs. 20,000, i.e. Rs. 8,000 has to be paid as upfront payment in
cash and thereafter monthly installment of Rs.1000 along with interest 1% per month has to be paid at the end of the month.
Mr. Ram Binod has purchased a television set by paying Rs. 8000 in cash on 2066.09.07. Installment of Magh and Falgun
has been paid on Falgun 05, 2066. By mentioning the relevant provision, determine the time of supply for Magh and Falgun
Month installments. 4

c) M/s Kiran Distributors is engaged in the wholesale business of various types of noodles. The firm is registered in VAT.
The firm has taken a loan of Rs. 10 lacs from M/s Saraswati Bank for purchasing a delivery van. Bank has paid the total
amount for the purchase of Van but has kept the ownership of van in the name of the bank till the loan is fully repaid by
Kiran Distributors. Kiran has to pay Rs. 25,000 per month towards the principal and interest to the Bank as part of their loan
agreement. Rs. 130,000 was paid as VAT during the purchase of van. By mentioning the relevant provision of the Act, can
Kiran Distributors claim the input tax credit? 4

d) ABC Ltd, a foreign party got a contract from an organization in Nepal. Before the start of the work, they have registered
themselves with VAT. As a tax professional, they enquire you on the provision for payment of VAT as per the VAT Act.
Advice ABC Ltd. 4

e) What is Debit / Credit Note? 4

Answer:

The Institute of Chartered Accountants of Nepal


33 of 62
Suggested Answers – Income Tax and VAT
CAP II Examination – December 2012
a) As per section 8(3) of the act, any person registered or not in Nepal engaged in constructing commercial buildings,
apartments, shopping malls or similar having value more than Rs.5 Million need to pay VAT on construction cost, if not paid
to registered person.
Further , Rule 6(kha) of the VAT Rules states that any person who, for commercial purposes, is constructing Building,
Apartment, Shopping Complex or similar as directed by the IRD amounting more than Rs.5 Million has to get it constructed
only from VAT registered person

In the referred case, since EFG Ltd. got the construction of a shopping complex for commercial purposes from non VAT
registered party, hence as per section 8(3) of the act, EFG Ltd. has to deposit the VAT on transaction value i.e. 13% of Rs.55
Lacs = Rs. 7.15 Lacs to the Inland Revenue. With regard to taxability of Garibdas Construction, since the amount of
transaction has exceeded minimum slab, it has to get registered itself in VAT.

b) As per section 6(3)(kha) of the VAT Act, in the case of contractual provision for paying the value of goods or services
partially in more than one day on an installment basis, earliest of due date as per the contract or actual payment date shall be
considered as Time of Supply.
In the month of Magh 2066, due date is Magh end but actual payment date is Falgun 05, hence in this case, time of supply is
Magh End.
For the month of Falgun 2066, due date is Falgun end but actual payment date is Falgun 05, hence in this case, time of supply
is Falgun 05.

c) As per section 17(5ka) of the act, if a tax payer has purchased or imported capital goods under a loan agreement within a
financial lease, then input tax paid can be claimed by the concerned tax payer.

In finance lease, all the benefits and risks are to be borne by the lessee, only legal passing of the title happens after the
stipulated date mentioned in the agreement. In this case, although the legal title still vests with the bank, but the firm can take
the input tax credit as per section 17(5ka).

d) Section 19 of the VAT Act states about the payment of value added tax as following:
1) A taxpayer shall have to pay the tax for each month within twenty five days of the close of the month.
2) If a taxpayer does not pay the tax within the time limit specified under (1), an extra charge of ten percent per annum shall
be imposed on the tax due.
3) If a taxpayer applies to the Director General for the exemption of the additional charges provided by (2) stating the reason
that the failure to make a timely payment was caused by extraordinary circumstances beyond the taxpayer's control, the
Director General may, if he finds the reason reasonable, exempt such charges.
4) The charges pursuant to (2) and and the interest pursuant to Section 26, shall be charged from the date on which the tax
first became due.
5) If a tax officer makes tax assessment under section 20 and finds that the amount of tax to be collected from the tax payer
in a tax period is less than the amount he is entitled for refund, then extra charge and interest shall not collected in that tax
period.
6) Tax can be paid through the Good for Payment Cheque issued by the Bank. In such a case, the day tax office receives the
good for payment cheque, it is deemed that the tax is collected.
7) Interest shall not be collected on Interest, Extra charge and penalties.

e) As per Rule 20 of the VAT Rules, a debit note/credit note is a document issued by a registered person owing to a change
in the value of the goods or services supplied by him previously.
The debit or credit note must include the following contents:
(a) Serial Number,
(b) Date of issue,
(c) Name, address and registration number of the supplier,
(d) Recipient's name, address, and registration number if he is a registered person,
(e) Number and date of the tax invoice connected with the transaction,
(f) Particulars of the goods or services and reason of credit or debit,
(g) Amount credited or debited,
(h) Tax amount credited or debited.
A registered taxpayer shall maintain a monthly record of credit or debit notes referred to in sub-rule (1)

6. Write on the following:


a) A lawyer practicing in Nepal has provided consultancy services to the Legal Firm situated in United Kingdom. The legal
firm situated in United Kingdom has used such services for the release of persons in USA. Where is place of supply of
services as per Value Added Tax Act/Rules? 5
b) There are some cases where Value Added Tax (VAT) paid on purchase is not allowed- no credit, even if output of
registered person is VAT attractive. Mention those cases as prescribed under Value Added Tax/Rules. 5
The Institute of Chartered Accountants of Nepal
34 of 62
Suggested Answers – Income Tax and VAT
CAP II Examination – December 2012
c) State the provisions on Value Added Tax (VAT) refund under Value Added Tax Act, 2052. 5
d) What are the records that are to be maintained by the tax payer as per rule 23 of VAT rules, 2053? 5

Answer:
a) Section 6 of Value Added Tax Act, 2052 deals with time and place of supply of goods or services.
1. For the purpose of assessment and collection of tax under this Act, the determination of the fact whether the supply of any
goods or services has taken place within or outside Nepal shall be as prescribed.
Further, as per rule 16 of Value Added Rules, 2053, the place of supply of a service shall be the place where the benefit of
that service is received.
In the case given in question, the lawyer in Nepal has provided service to Legal Firm in United Kingdom, the Legal Firm of
United Kingdom in turn has used such services and the payment of such services has been made from United Kingdom to
Nepal, and hence the place of supply is United Kingdom.

b) For the purpose of section 17 of Value Added Tax Act, 2052 following are the goods or services wherein VAT paid on
purchase is not allowed-no credit, as per rule 41 of Value Added Rules, 2053.
i. Consumption of drinkable items (soft drinks, water juice or similar)
ii. Liquor items (beer, wine, whiskey or similar)
iii. Petrol for vehicles used for human transport
iv. Expenses on Entertainment

c) Following are the provisions related to Value Added Tax (VAT) refund under Value Added Tax Act, 2052;
1. According to Section 24(3), if a registered person has continues credit of VAT for six months, credit is eligible for VAT
refund. In case any registered person has export during the month at least 40% of total sales in that month, the credit is
eligible for VAT refund as per section 24(4).
2. According to section 25, VAT paid by following person or paid for following event may be refunded, upon request for
refund within 3 years from the date of transaction on which the claim for refund is based;
i. Diplomat, privileged on a reciprocal basis from Ministry of Foreign Affairs, person engaged in Regional or International
Organization or missions having diplomatic privileges. This refund shall not be allowed for diplomats for purchase of goods
or services at a time for less than Rs. 1,500 as per section 15 (1ka).
ii. Institution or VAT paid such institution on which Ministry of Finance, has granted the privileges of tax exemption
iii. Tax exemption project by Ministry of Finance under bilateral and multilateral agreement
iv. Any tax collected by mistake.

3. Foreign tourist visiting in Nepal, if purchase and take away from Nepal via air transport shall get refund VAT on those
assets, if the cost paid is higher than Rs. 15,000. A service charge of 3% of refund is charged on refund (section 25 ka).

d) As per rule 23 of VAT rules, a registered person shall for the purpose of the Act and these Rules maintain records of the
following information, documents and details:-
 Information as per Schedule -7.
 Records relating to trade, accounts, cash receipts and payments.
 Tax invoices and abbreviated tax invoices issued.
 Tax invoices and abbreviated tax invoices received.
 All documents relating to his imports and exports,
 All debit and credit notes.
 Books of purchases and sales as per Schedules 8 and 9.

The Institute of Chartered Accountants of Nepal


35 of 62

You might also like